Sunteți pe pagina 1din 36

BASIC CONCEPTS List price, trade discount, prepaid freight, cash discount

Revenue Cycle 4. On June 1, Pitt Corp. sold merchandise with a list price of $5,000 to Burr on account. Pitt
Trade discounts allowed trade discounts of 30% and 20%. Credit terms were 2/15, n/40 and the sale was
1. Melissa Company sold an item on credit for P5,000,000 less multiple trade discounts of 20 made FOB shipping point. Pitt prepaid $200 of delivery costs for Burr as an accommodation.
and 5. Which entry to record this sale is correct? On June 12, Pitt received from Burr a remittance in full payment amounting to:
A. Debit Accounts Receivable for P4,125,000 and credit Sales for P3,750,000 and Output A. $2,744 C. $2,944
Tax for P375,000. B. $2,940 D. $3,140 Becker 2013
B. Debit Accounts Receivable for P4,180,000 and credit Sales for P3,800,000 and Output
Tax for P380,000. 2. On June 1, 2012, Jeanah Company sold merchandise with a list price of P5,000,000 to a
C. Debit Accounts Receivable for P5,500,000 and credit Sales for P5,000,000 and Output customer. The entity allowed trade discounts of 20% and 10%. Credit terms were 5/10, n/30
Tax for P500,000. and the sale was made FOB shipping point. The entity prepaid P100,000 of delivery cost for
D. Debit Accounts Receivable for P5,500,000 and Trade Discount for P1,200,000 and credit the customer as an accommodation. On June 11, 2012, what is the full remittance from the
Sales for P5,000,000, Output Tax for P500,000 and Allowance for Sales Discount for customer? (M1)
P1,200,000. CPAR 0503 A. 3,420,000 C. 3,600,000
B. 3,520,000 D. 3,700,000 CPAR 1012
2. Bernadette Company sold an item on credit for P5,000,000 less multiple trade discounts of
20 and 5. The correct entry to record this sale is 5. On June 1, 2014 Compassion Company sold merchandise with a list price of P 1,000,000 to
A. Debit Accounts Receivable for P4,180,000 and credit Sales for P3,800,000 and Output a customer. The entity allowed trade discounts of 20% and 10%. Credit terms were 5/10,
tax for P380,000. n/30 and the sale was made FOB shipping point. The entity prepaid P50,000 of delivery cost
B. Debit Accounts Receivable for P5,500,000 and credit Sales for P5,000,000 and Output for the customer as an accommodation On June 11, 2014, what amount is received from the
Tax for P600,000. customer as full remittance?
C. Debit Accounts Receivable for P4,300,000 and Sales Discount for P1,200,000 and credit A. 684,000 C. 770,000
Sates for P5,000,000 and Output Tax for P500,000. B. 720,000 D. 794,000 FA © 2014
D. Debit Accounts receivable for P5,500,000 and Trade Discount for P1,200,000 and credit
Sales for P5,000,000, Output Tax for P500,000 and Allowance for Sales Discount for 35. Crabtree & Evelyn Corp. normally sells ladies’ apparels to preferred customers on account.
P1,200,000. CPAR 1004 Trade discounts of 20% and 10% are granted with credit terms of 2/15, n/60. On August 1,
2004, apparels with a list price of P250,000 were sold FOB seller’s warehouse. Crabtree &
Accounts Receivable Evelyn prepaid the freight of P150 as an accommodation.
3. The following data are given for Mazda, Inc.: The amount that Crabtree & Evelyn should receive on August 10, 2004 when the customer
Cash Credit Total remits the full payment is
Cost of sales P50,000 P450,000 P500,000 A. P171,603 C. P176,506
Cash received from customers 65,000 585,000 650,000 B. P171,650 D. P176,550 CRC 0504
Assuming merchandise was marked to sell as follows: Cash sales, at 30% about cost and
credit sales, at 40% above cost, all of which are collectible. The balance of accounts 15. On April 28, 2012 Malcom Company sold merchandise with a list price of P5,000,000 to
receivable at the end of the period was (M) Forbes. Malcom allowed trade discounts of 20% and 10%. Credit terms were 5/10, n/30.
A. P12,500 C. P135,000 The goods were shipped FOB destination, freight collect. Total freight charges paid by the
B. P45,000 D. P147,500 R&E 2012 Forbes amounted to P50,000. On May 8, 2012, Malcom received from Forbes full remittance
of (M1)
Collection A. P3,370,000 C. P3,550,000
B. P3,420,000 D. P3,600,000 Cabarles A. $14,000 C. $15,600
B. $14,400 D. $20,000 AICPA 0590
Purchases – Gross Method
Inventoriable costs 11. MORO Co. purchased an item of merchandise quoted and listed at P150,000 under the
List price, trade discount following terms: Trade discounts of 15%, 10% and 5%; 2/10, n/30. What was the invoice
6. Ami Retailers purchased merchandise with a list price of $100,000, subject to a trade price of the merchandise? (E)
discount of 20 percent and credit terms of 2/10, n/30. At what amount should Ami record the A. P105,000.00 C. P100,842.00
cost of this merchandise if the gross method is used? (E) B. P109,012.50 D. P104,695.60 RPCPA 1087
A. $100,000 C. $98,000
B. $80,000 D. $78,400 S, S & S 12. Kindness Company regularly buys sweaters and is allowed a trade discount of 20% and 10%.
The entity made a purchase on March 20 and received an invoice with a list price of
9. Walters Co. purchased raw materials with a catalog price of $70,000 on March 2, 2010. P900,000, a freight charge of P50,000, and payment terms of net 30 days. The entity should
Credit terms of 4/20, n/60 applied. Walters uses a perpetual inventory system and the gross record the purchase at what amount?
price method. If Walters pays for the purchase on March 18, 2010, calculate what amount is A. 630,000 C. 680,000
recorded for inventory on March 2, 2010. B. 648,000 D. 698,000 FA © 2014
A. $42,000 C. $67,200
B. $56,000 D. $70,000 NB&J 11e List price, trade discount, freight-in
13. Dean Sportswear regularly buys sweaters from Mill Company and is allowed trade discounts
7. West Retailers purchased merchandise with a list price of $20,000, subject to trade discounts of 20% and 10% from the list price. Dean made a purchase on March 20, 2004, and receive
of 20% and 10%, with no cash discounts allowable. West should record the cost of this an invoice with a list price of P600,000, a freight charge of P15,000 and payment terms of
merchandise as (E) 2/10, n/30. Dean should record the purchase at (E)
A. $14,000 C. $15,600 A. 432,000 C. 438,360
B. $14,400 D. $20,000 AICPA 0590 B. 447,000 D. 435,000 AICPA

8. Walsh Retailers purchased merchandise with a list price of $50,000, subject to trade 14. Galleria Sportswear, Inc. regularly buys sweaters from Bon Company and is allowed a trade
discounts of 20% and 10%, with no cash discounts allowable. Walsh should record the cost discount of 20% and 10% from the list price. Galleria made a purchase on March 20, 2012,
of this merchandise as and received an invoice with a list price of P90,000, a freight charge of P5,000, and payment
A. $35,000. C. $39,000. terms of net 30 days. What is the total cost of the inventory purchase? (E)
B. $36,000. D. $50,000. AICPA adapted A. P63,000 C. 69,000
B. P64,000 D. P69,800 R&E 2012
9. Utley Retailers purchased merchandise with a list price of $30,000, subject to trade discounts
of 20% and 10%, with no cash discounts allowable. Utley should record the cost of this 81. Ella Company regularly buys sweaters from Millard Company and is allowed trade discount of
merchandise as (E) 20% and 10% from a list price. Ella made a purchase on March 20 and received an invoice
A. $21,000. C. $23,400. with a list price of P4,000,000, a freight charge of P100,000, and payment terms of net 30
B. $21,600. D. $30,000. K, W & W days. Ella should record the purchase at (E)
A. 2,880,000 C. 4,000,000
10. West Retailers purchased merchandise with a list price of $20,000, subject to trade discounts B. 2,980,000 D. 4,100,000 Siy
of 20% and 10%, with no cash discounts allowable. West should record the cost of this
merchandise as (E) List price, trade discount, freight, insurance, import-related costs
15. Brilliant Company purchased motorcycles from various countries for export to other countries. Shipping 1,500
The entity has incurred the following costs during the current year: Special handling charges 2,000
Cost of purchases based on vendors' invoices 5,000,000 These goods were received on December 31, 2011. In Kerr’s December 31, 2011 balance
Trade discounts on purchases already deducted from vendors' invoices 500,000 sheet, what amount of cost for these goods should be included in inventory?
Import duties 400,000 A. $54,500 C. $52,000
Freight and insurance on purchases 1,000,000 B. $53,500 D. $50,000 AICPA 1191
Other handling costs relating to imports 100,000
Salaries of accounting department 600,000 Invoice price
Brokerage commission paid to agents for arranging imports 200,000 19. The following costs were among those incurred by Woodcroft Corporation during 2011:
Sales commission paid to sales agents 300,000 Merchandise purchased for resale $500,000
After-sales warranty costs 250,000 Salesmen's commissions 40,000
What is the total cost of the purchases? Interest on notes payable to vendors 5,000
A. 5,700,000 C. 6,500,000 How much should be charged to the cost of the merchandise purchases? (E)
B. 6,100,000 D. 6,700,000 FA © 2014 A: $500,000 C: $540,000
B: $505,000 D: $545
FOB Shipping point
16. An enterprise with a December 31 year-end purchased 2,000 of inventory on account. The 7. On December 28, 1996, Arlington Co. purchased goods costing P1,000,000. The terms were
seller was responsible for delivery to the shipping point, with freight of 50 paid at destination FOB destination. Some of the cost incurred in connection with the sale and delivery of the
by the buyer. The invoice date was December 27, 2001, and the goods arrived on January 3, foods were as follows:
2002. What is the correct amount of inventory and freight-in, respectively, relating to this Packaging for shipment P20,000
purchase on the 2001 financial statements? Shipping 30,000
CIA 0592 A. B. C. D. Special handling charges 40,000
Inventory 0 2,050 0 2,000 These goods were received on December 31, 1996.
Freight-In 0 0 50 50 In Arlington’s December 31, 1996 balance sheet, what amount of costs for these goods
should be included in inventory? (E))
FOB destination A. P1,040,000 C.
17. An enterprise with a December 31 year-end purchased 2,000 of inventory on account. The P1,090,000
terms required the seller to deliver to the destination, with freight of 50 paid at destination by B. P1,070,000 D. P1,0
the buyer. The invoice date was December 27, 2001, and the goods arrived on January 3,
2002. What is the correct amount of inventory and freight-in relating to this purchase on the 20. On December 28, 2001, Nord Manufacturing Co. purchased goods costing $50,000. The
2001 financial statements? terms were FOB destination. Some costs incurred in connection with the sale and delivery of
CIA 0592 IV-33 A. B. C. D. the goods were
Packaging for shipping $1,000
Inventory 0 2,050 0 2,000
Shipping 1,500
Freight-In 0 0 50 50
Special handling charges 2,000
These goods were received on December 31, 2001. In Nord’s December 31, 2001 balance
18. On December 28, 2011, Kerr Manufacturing Co. purchased goods costing $50,000. The sheet, what amount of cost should be included in inventory? (E)
terms were FOB destination. Some of the costs incurred in connection with the sale and A. $54,500 C. $52,000
delivery of the goods were as follows: B. $53,500 D. $50,000 AICPA 1191
Packaging for shipment $1,000
Freight-out 5,000
21. On December 26, 2004, Karen Company purchased goods costing P5,000,000. The freight Purchase returns 2,000
term is FOB destination. Some of the costs incurred in connection with the sale and delivery if Howe's 2010 inventoriable cost was
the goods were: A. $300,000. C. $306,000.
Packaging for shipment 100,000 B. $303,000. D. $311,000. AICPA adapted
Shipping 200,000
Special handling charges 300,000 95. The following information applied to Michaels Company for 2014:
These goods were received on December 31, 2004. In the December 31, 2004 balance sheet, Merchandise purchased for resale ..................... $400,000
what amount of cost for these goods should be included in inventory? (E) Freight-in ........................................... 7,500
A. 5,000,000 C. 5,300,000 Interest on notes payable to vendors ................. 3,000
B. 5,600,000 D. 5,500,000 CPAR Purchase returns ..................................... 2,500
Michaels' inventoriable cost for 2014 was
Purchases, freight-in A. $405,000 C. $407,500.
22. On December 15, 2011, Flanagan purchased goods costing $100,000. The terms were FOB B. $406,000. D. $409,000. S&S 19e
shipping point. Costs incurred by Flanagan in connection with the purchase and delivery of
the goods were as follows: 24. The following information applied to Fenn, Inc. for 1989:
Normal freight charges $3,000 Merchandise purchased for resale $400,000
Handling costs 2,000 Freight-in 10,000
Insurance on shipment 500 Freight-out 5,000
Abnormal freight charges for express shipping 1,200 Purchase return 2,000
The goods were received on December 17, 2011. What is the amount that Flanagan should Fenn’s 1989 inventoriable cost was (E)
charge to inventory and to current period expense? A. $400,000 C. $408,000
Wiley 2012 A B C D B. $403,000 D. $413,000 AICPA 1190
Inventory $3,000 $5,000 $5,500 $6,700
Current period expense $3,700 $1,700 $1,200 $0 25. The following information applied to Atlas Co. for 2001:
Merchandise purchased for resale $800,000
Purchases, Freight-in, Purchase Return Freight-in 20,000
57. The following information applied to Greer, Inc. for 2001: Freight-out 10,000
Merchandise purchased for resale $200,000 Purchase returns 4,000
Freight-in 8,000 The company’s 2001 inventoriable cost was (E)
Freight-out 5,000 A. $800,000 C. $816,000
Purchase returns 2,000 B. $806,000 D. $826,000 AICPA 1190
Greer's 2001 inventoriable cost was (E)
A. $200,000. C. $206,000. *. The following information pertains to Rasner Company for 2009:
B. $203,000. D. $211,000. K, W & W Merchandise purchased for resale 4,000,000
Freight out 200,000
23. The following information applied to Howe, Inc. for 2010: Freight in 500,000
Merchandise purchased for resale $300,000 Storage cost 50,000
Freight-in 8,000 Purchase returns 120,000
The inventoriable cost should be Insurance cost during transit of purchased goods 100
A. 4,250,000 C. 4,500,000 Total $4,925
B. 4,380,000 D. 4,630,000 Siy What amount should Den record as the cost of inventory as a result of this purchase?
A: $3,925 C: $4,825
95. The following information applied to Landon Company for 2005: B: $4,650 D: $4,925 AICPA R08
Merchandise purchased for resale ..................... $300,000
Freight-in ........................................... 7,500 Importation charges
Interest on notes payable to vendors ................. 3,000 27. Brilliant Company purchased motorcycles from various countries for export to other countries.
Purchase returns ..................................... 1,500 The entity has incurred the following costs during the current year:
Landon's inventoriable cost for 2005 was (E) Cost of purchases based on vendors' invoices 5,000,000
A. $309,000. C. $306,000. Trade discounts on purchases already deducted from vendors' invoices 500,000
B. $307,500. D. $301,500. S, S & S Import duties 400,000
Freight and insurance on purchases 1,000,000
95. The following information applied to Michaels Company for 2014: Other handling costs relating to imports 100,000
Merchandise purchased for resale ..................... $400,000 Salaries of accounting department 600,000
Freight-in ........................................... 7,500 Brokerage commission paid to agents for arranging imports 200,000
Interest on notes payable to vendors ................. 3,000 Sales commission paid to sales agents 300,000
Purchase returns ..................................... 2,500 After-sales warranty costs 250,000
Michaels' inventoriable cost for 2014 was (M) What is the total cost of the purchases?
A. $409,000. C. $406,000. A. 5,700,000 C. 6,500,000
B. $407,500. D. $405,000 S&S 18e B. 6,100,000 D. 6,700,000 FA © 2014

Purchases, freight-in, handling costs, insurance Cash payments


25. December 15, 2011, Boston purchased goods costing P100,000. The terms were FOB Payment within discount period
shipping point. Costs incurred by Boston in connection with the purchase and delivery of the 28. A firm purchased goods with a purchase price of $1,000 and credit terms of 1/10 net 30. The
goods were as follows: firm paid for these goods on the 5th day after the date of sale. The firm must pay _____ for
Normal freight charges P3,000 the goods. (E)
Handling costs 2,000 A. $990 C. $1,000
Insurance on shipment 500 B. $900 D. $1,100 Gitman
Abnormal freight charges for express shipping 1,200
What is the total amount to be charged to inventory? (E) 29. A firm purchased goods on January 27 with a purchase price of $1,000 and credit terms of
A. P103,000 C. P105,500 2/10 net 30 EOM. The firm paid for these goods on February 9. The firm must pay _____ for
B. P105,000 D. P106,700 Cabarles the goods. (E)
A. $1,000 C. $800
Invoice Price, Freight-in, Insurance, Materials & Labor B. $980 D. $900 Gitman
26. The following costs pertain to Den Co.’s purchase of inventory:
700 units of product A $3,750 Trade discount, payment within discount period
Freight-in 175 30. The list price and term of a merchandise purchased is P8,000 less 15%, 10%, 5%, 2/10,
Cost of materials and labor incurred to bring product A to saleable condition 900 n/30. The payment within the discount period is (E)
A. P5,697.72 C. P5,488. 34. Sylvia Systems Corp. received an invoice for P12,000 from Lyndon Enterprises Co. for a
B. P5,654.00 D. P7,840 RPCPA 0587 shipment of specialty supplies, F.O.B. destinations, terms 2/10, n/60. In addition, the Sylvia
Systems paid P800 for freight on this shipment to a trucking firm. If the invoice from Lyndon
31. MORO Co. purchased an item of merchandise quoted and listed at P150,000 under the Enterprises is paid 15 days after it is received, Sylvia Systems must pay (E)
following terms: Trade discounts of 15%, 10% and 5%; 2/10, n/30. How much was the A. P10,960 D. P10,976
payment if paid within the discount period? (E) B. P11,200 E. P12,544
A. P102,900.00 C. P100,842.00 C. P11,184 RPCPA 1078
B. P106,832.25 D. P104,695.60 RPCPA 1087
Trade discount, FOB shipping point, prepaid freight, payment within discount period
32. Celine Company purchased an item of merchandise quoted and listed at P150,000 under the 26. On June 1, 1995, Orange Corp. sold merchandise with list price of P5,000 to Mont on
following terms: Trade discounts of 15%, 10%, 5%, 2/10, n/30. account. Orange allowed trade discounts of 30% and 20%. Credit terms were 2/15, n/40
How much was the cash payment if settlement was made within the discount period? and the sale was made FOB shipping point. Orange prepaid P200 of delivery costs for Mont
A. P106,832.25 C. P102,900.00 as an accommodation. On June 12, 1995, Orange received from Month a remittance in full
B. P104,693.50 D. P100,842.00 R&E 2012 payment amounting to (M)
A. P2,744 C. P3,140
Trade discount, FOB shipping point, prepaid freight, cash discount, B. P2,940 D. P2,944 RPCPA 0596
45. On June 1, 1997, Ebecom Corp. sold merchandise with a list price of P200,000 to
Greenwich, Inc. on account. Ebecom allowed trade discounts of 20% and 10%. Credit terms Purchases – Net Method
were 2/30, n/60 and the sale was made FOB shipping point. Ebecom prepaid P3,000 of Accounts payable
insurance for Greenwich as an accommodation. 35. On December 28 of the current year, the Baxter Company purchased $30,000 in
On June 20, 1997, Greenwich paid Ebecom his full account of (E) merchandise on terms of 3/15, n/45. Baxter immediately returned 20 percent of this
A. P150,060 C. P147,060 merchandise because it was the wrong color. If the company records its purchases net of any
B. P144,120 D. P141,120 RPCPA 0597 discounts, what amount of liability should be reported until payment is made?
A $20,420 C $23,400
33. On June 1, 2011, Pitt Corp. sold merchandise with a list price of $5,000 to Burr on account. B $23,280 D $24,000
Pitt allowed trade discounts of 30% and 20%. Credit terms were 2/15, n/40 and the sale was
made FOB shipping point. Pitt prepaid $200 of delivery costs for Burr as an accommodation. Inventoriable costs
On June 12, 2011, Pitt received from Burr a remittance in full payment amounting to Trade discount, cash discount
A. $2,744 C. $2,944 36. Freiberg Co. regularly buys shirts from Halberstadt Company and is allowed trade discounts
B. $2,940 D. $3,140 Wiley 2011 of 20% and 10% from the list price. Freiberg purchased shirts from Halberstadt on May 27,
1996 and received an invoice with a list price of P100,000 and payment terms 2/10, n/30.
84. On June 1, 2009, Elijah Company sold merchandise with a list price of P5,000,000 to XYZ. Freiberg should record the purchase, using the net method, at (E)
Elijah allowed trade discounts of 20% and 10%. Credit terms were 5/10, n/30 and the sale A. P68,600 C. P70,560
was Made FOB shipping point. Elijah prepaid P50,000 of delivery cost for XYZ as an B. P72,000 D. P70,000 RPCPA 1097
accommodation. On June 11, 2009, Elijah received from XYZ full remittance of
A. 3,420,000 C. 3,800,000 37. Dixon Menswear Shop regularly buys shirts from Colt Company and is allowed trade
B. 3,470,000 D. 3,850,000 Siy discounts of 20% and 10% from the list price. Dixon purchased shirts from Colt on May 27,
2011, and received an invoice with a list price amount of $5,000, and payment terms of 2/10,
FOB destination, freight paid by buyer, payment after the discount period n/30. Dixon uses the net method to record purchases. Dixon should record the purchase at
A: $3,600 C: $3,500 41. Pye Co. records purchases at net amounts. On May 5 Pye purchased merchandise on
B: $3,528 D: $3,430 Wiley 2011 account, $8,000, terms 2/10, n/30. Pye returned $500 of the May 5 purchase, and received
credit on account. At May 31 the balance had not been paid. The amount to be recorded as
Cash discount a purchase return is (M)
38. On September 1, Hydra purchased $9,500 of inventory items on credit with the terms 1/15, A. $450. C. $500.
net 30, FOB destination. Freight charges were $200. Payment for the purchase was made on B. $510. D. $490. K, W & W
September 18. Assuming Hydra uses the perpetual inventory system and the net method of
accounting for purchase discounts, what amount is recorded as accounts payable from this Purchase discount taken
purchase? (E**) 13. Walters Co. purchased raw materials with a catalog price of $70,000 on March 2, 2010.
A. $9,405. C. $9,700. Credit terms of 4/20, n/60 applied. Walters uses a perpetual inventory system and the net
B. $9,605. D. $9,500. KW&W 1e price method. If Walters pays for the purchase on March 18, 2010, what amount is recorded
in the purchase discounts taken account?
39. Alison's dress shop buys dresses from McGuire Manufacturing. Alison purchased dresses A. $ 0 C. $14,000
from McGuire on July 17, and received an invoice with a list price amount of $6,000 and B. $ 2,800 D. $28,000 NB&J 11e
payment terms of 2/10, n/30. Alison uses the net method to record purchases. Alison should
record the purchase at: (E) Purchase discount lost
A. $5,940. C. $6,000. Invoice price
B. $5,880. D. $6,120. S&S 6e 14. Walters Co. purchased raw materials with a catalog price of $70,000 on March 2, 2010.
Credit terms of 4/20, n/60 applied. Walters uses a perpetual inventory system and the net
16. Phillips Corp. purchased raw materials with a catalog price of $60,000. Credit terms of 3/15, price method. If Walters pays for the purchase on March 31, 2010, what amount is recorded
n/60 apply. If Phillips uses the net price method, the purchase should be recorded at (E) in the purchase discounts lost account?
A. $61,800 C. $58,200 A. $ 0 C. $14,000
B. $60,000 D. $51,000 NB&J 11e B. $ 2,800 D. $28,000 NB&J 11e

8. Walters Co. purchased raw materials with a catalog price of $70,000 on March 2, 2010. Invoice price, purchase return
Credit terms of 4/20, n/60 applied. Walters uses a perpetual inventory system and the net 41. Winsor Co. records purchases at net amounts. On May 5 Winsor purchased merchandise on
price method. If Walters pays for the purchase on March 18, 2010, calculate what amount is account, $16,000, terms 2/10, n/30. Winsor returned $1,200 of the May 5 purchase and
recorded for inventory on March 2, 2010. (E) received credit on account. At May 31 the balance had not been paid. By how much should
A. $42,000 C. $67,200 the account payable be adjusted on May 31?
B. $56,000 D. $70,000 NB&J 11e A. $0. C. $320.
B. $344. D. $296. KW&W 1e
Purchase return
40. Winsor Co. records purchases at net amounts. On May 5 Winsor purchased merchandise on 42. Pye Co. records purchases at net amounts. On May 5 Pye purchased merchandise on
account, $16,000, terms 2/10, n/30. Winsor returned $1,200 of the May 5 purchase and account, $8,000, terms 2/10, n/30. Pye returned $500 of the May 5 purchase, and received
received credit on account. At May 31 the balance had not been paid. The amount to be credit on account. At May 31 the balance had not been paid. By how much should the
recorded as a purchase return is (M) account payable be adjusted on May 31? (M)
A. $1,080. C. $1,200. A. $0. C. $160
B. $1,224. D. $1,176. KW&W 1e B. $170 D. $150 K, W & W
Total purchases, purchase returns, discounts taken 45. On July 1, Clio Company recorded purchases of inventory of $40,000 and $50,000 under
42. DATACORP a computer store in Virra Mall, Greenhills specializes in the sale of IBM credit terms of 2/15, net 30. The payment due on the $40,000 purchase was remitted on July
compatibles and software packages and had the following transactions with one of its 14. The payment due on the $50,000 purchase was remitted on July 25. Under the net
suppliers: method and the gross method, these purchases should be included at what respective net
Purchase of IBM compatibles P328,000 amounts in the determination of cost of goods available for sale? (M)
Purchases of commercial software packages 90,000 Gleim A. B. C. D.
Returns and allowances 8,000 Net Method $90,000 $89,200 $88,200 $88,200
Purchases discounts taken 2,700 Gross Method $90,000 $88,200 $89,200 $88,200
Purchases were made throughout the year on terms 3/10, n/60. All returns and allowances
tool place within 5 days of purchase and prior to any payment of account. (M) 75. On August 1, Stephan Company recorded purchases of inventory of $80,000 and $100,000
Discount lost is under credit terms of 2/15, net 30. The payment due on the $80,000 purchase was remitted
A. P9,840 C. P6,900 on August 14. The payment due on the $100,000 purchase was remitted on August 29.
B. P9,600 D. P7,140 RPCPA 1093 Under the net method and the gross method, these purchases should be included at what
respective net amounts in the determination of cost of goods available for sale? (M)
43. Rock Distributors, a computer store in Virra Mall S, S & S A. B. C. D.
Purchases of IBM compatibles P3,280,000 Net Method $178,400 $176,400 $176,400 $180,000
Purchases of commercial software package 900,000 Gross Method $176,400 $176,400 $178,400 $176,400
Returns and allowances 80,000
Purchase discounts taken 27,000 Cost of Goods Available for Sale
Purchases were made throughout the year on terms 3/10, n/30. All returns and allowances 6. On July 1, 2012, Irene Company recorded purchases of inventory of P3,000,000 and
took place within 5 days of purchase and prior to any payment of account. How much were P2,000,000 under credit terms of 2/15, net 30. The payment due on the P3,000,000 purchase
the discounts lost? (M1) was remitted on July 16. The payment due on the P2,000,000 purchase was remitted on July
A. P98,400 C. P71,400 31. Under the gross method and net method, respectively, these purchases should be
B. P96,000 D. P69,000 R&E 2012 included at what amount in the determination of cost of goods available for sale?
A. 4,940,000 and 4,900,000 C. 4,900,000 and 5,000,000
44. Duke company specializes in the sale of IBM compatibles and software packages. It had the B. 4,900,000 and 4,940,000 D. 5,000,000 and 4,900,000 CPAR 1012
following transactions with one of its suppliers:
Purchases of IBM compatibles 1,700,000 83. On August 1, Erica Company recorded purchases of inventory of P800,000 and P1,000,000
Purchases of commercial software packages 1,200,000 under credit terms of 2/15, net 30. The payment due on the P800,000 purchase was remitted
Returns and allowances 50,000 on August 14. The payment due on the P1,000,000 purchase was remitted on August 29.
Purchase discounts taken 17,000 Under the net method and the gross method, these purchases should be included at what
Purchases were made throughout the year on terms 2/10, n/30. All returns and allowances respective net amounts in the determination of cost of goods available for sale?
took place within 5 days of purchase and prior to any payment on account. Discount lost is Siy A B C D
(M1)
Net Method 1,764,000 1,764,000 1,784,000 1,800,000
A. 57,000 C. 17,000
Gross Method 1,784,000 1,800,000 1,764,000 1,764,000
B. 40,000 D. 41,000 RPCPA
Cost of Goods Sold
Purchases - Gross Method vs. Net Method
6. Catapult Corp. purchased merchandise during 2012 on credit for P200,000; terms 2/10, n/30.
Purchases
All of the gross liability except P40,000 was paid within the discount period. The remainder
was paid within the 30-day term. At the end of the annual accounting period, December 31, Accounts payable 50,000
2012, 90% of the merchandise had been sold and 10% remained in inventory. The entity has What is Saddle's accounts payable balance as of September 30, Year 2, after the
no beginning inventory. The entity uses net method of recording purchases. conversion?
If the entity used the gross method of recording purchases instead of the net method, the A. $49,200 C. $47,900
reported cost of goods sold would have been B. $49,100 D. $47,800 AICPA 1192
A. The same C. Lower by P720
B. Higher by P720 D. P176,400 Cabarles 48. Rabb Co. records its purchases at gross amounts but wishes to change to recording
purchases net of purchase discounts. Discounts available on purchases recorded from
Change from Gross Method to Net Method October 1, 1999 to September 30, 2000 totaled $2,000. Of this amount, $200 is still available
46. Andy Company records its purchases at gross but wishes to change to recording purchases in the accounts payable balance. The balances in Rabb’s accounts as of and for the year
at net. Discounts available on purchases recorded from October 1, 2002 to September 30, ended September 30, 2000 before conversion are
2003 totaled P400,000. Of this amount P50,000 is still available in the accounts payable Purchases $100,000
balance. The balances in the accounts before conversion are: Purchase discounts taken 800
Purchases 8,000,000 Accounts payable 30,000
Purchase discounts taken 100,000 What is Rabb’s accounts payable balance as of September 30, 2000 after the conversion?
Accounts payable 3,000,000 A. $29,800 C. $28,800
The adjusted accounts payable on September 30, 2003 should be B. $29,200 D. $28,200 AICPA 1192
A. 2,950,000 C. 2,600,000
B. 2,900,000 D. 3,000,000 CPAR 49. Rabb Company records its purchases at gross amount but wishes to change to recording
purchases net of purchase discounts. Discount available on purchases recorded from
2. Danielle Company records its purchases at gross amount but wishes to change to recording October 1, 2004, to September 30,2005, totaled P100,000. Of this amount, P10,000 is still
purchases net of discounts. Discounts available on purchases recorded from October 1, 2008 available in the accounts payable balance. The balances in the accounts as of and for the
to September 30, 2009, totaled P200,000. Of this amount, P50,000 is still available in the year ended September 30, 2005, before conversion are:
accounts payable balance. The balances in the accounts as of and for the year ended Purchases 5,000,000
September 30, 2009, before conversion are: Purchase discounts taken 40,000
Purchases 8,500,000 Accounts payable 1,500,000
Purchase discounts taken 80,000 What is the accounts payable balance as of September 30, 2005, after the conversion? (M)
Accounts payable 3,600,000 A. 1,490,000 C. 1,440,000
What is the accounts payable balance as of September 30, 2009, after conversion? B. 1,460,000 D. 1,410,000 AICPA
A. 3,400,000 C. 3,520,000
B. 3,470,000 D. 3,550,000 Siy TRIAL BALANCE
Difference between Debit & Credit Total
47. Saddle Co. records its purchases at gross amounts but wishes to change to recording 50. The following errors were made in preparing a trial balance: the $1,350 balance of Inventory
purchases net of purchase discounts. Discounts available on purchases recorded from was omitted; the $450 balance of Prepaid Insurance was listed as a credit; and the $300
October 1, Year 1 to September 30, Year 2 totaled $3,000. Of this amount, $800 is still balance of Salaries Expense was listed as Utilities Expense. The debit and credit totals of the
available in the accounts payable balance. The balances in Saddle's accounts, before trial balance would differ by
conversion, for the year ended September 30, Year 2 are A. $1,350. C. $2,100.
Purchases $200,000 B. $1,800. D. $2,250. S, S & S
Purchase discounts taken 900
zzz Your examination disclosed the following:
PROFIT & LOSS STATEMENT Merchandise inventory at December 31, 1982 amounted to P130,000 and gross profit rate is
Gross Sales 20% on net sales.
Gross profit, total purchases, ending inventory given Gross sales for 1982 amounted to
51. Reyes Company had a gross profit of $360,000, total purchases of $420,000, and an ending A. P68,000 D. P63,000
inventory of $240,000 in its first year of operations as a retailer. Reyes’s sales in its first year B. P68,750 E. P69,500
must have been C. P69,000 RPCPA 1083
A. $540,000. C. $180,000.
B. $660,000. D. $600,000. KW&W 1e Net Sales
Gross sales, sales returns
Common size income statement 53. Fenn Stores, Inc. had sales of $1,000,000 during December. Experience has shown that
52. A company has a 50% gross margin, general and administrative expenses of $50, interest merchandise equaling 7% of sales will be returned within 30 days and an additional 3% will
expense of $20, and net income of $10 for the year just ended. If the corporate tax rate is be returned within 90 days. Returned merchandise is readily resalable. In addition,
50%, the level of sales revenue for the year just ended was (E) merchandise equaling 15% of sales will be exchanged for merchandise of equal or greater
A. $90 C. $150 value. What amount should Fenn report for net sales in its income statement for the month of
B. $135 D. $180 CIA 1194, 0596 December?
A. $750,000 C. $850,000
1. The expenses other than interest expense of Sydney Company for the current year is 40% of B. $780,000 D. $900,000 Becker 13
cost of sales but only 20% of sales. Interest expense is 5% of sales. The amount of
purchases is 120% of cost of sales. Ending inventory is twice as much as the beginning Gross sales, sales returns & allowances, sales discount
inventory. The income after tax of 35% for the current year is P325,000. What is the amount 47. Neptune Company's gross sales in 2004 were $3,930,000. Assuming sales returns and
of sales for the current year? allowances were $74,000, sales discounts were $35,000, and freight-out was $28,000, what
A. 1,300,000 C. 2,000,000 were Neptune's net sales in 2004? (E)
B. 1,625,000 D. 2,500,000 Siy A. $3,793,000 C. $3,856,000
B. $3,821,000 D. $3,930,000 S&S 19e
5. The operating expenses other than interest expense of Jellie Company for the current year
amount to 40% of cost of sales but only 20% of sales. Interest expense is 5% of sales. The 54. Gross billings for merchandise sold by Lang Company to its customers last year amounted to
amount of purchases is 120% of cost of sales. Ending inventory is twice as much as the $15,720,000; sales returns and allowances were $370,000, sales discounts were $175,000,
beginning inventory. The income after tax of 30% for the current year is P560,000. What is and freight-out was $140,000. Net sales last year for Lang Company were (E)
the amount of sales for the current year? A. $15,720,000. C. $15,175,000.
A. 1,485,000 C. 2,285,000 B. $15,350,000. D. $15,035,000. KW&W 1e
B. 2,080,000 D. 3,200,000 CPAR 1012
27. Gross billings for merchandise sold by Pye Company to its customers last year amounted to
8. December 31, 1982 balances of selected accounts of Rachelle Co. and pertinent information $12,720,000; sales returns and allowances were $270,000, sales discounts were $175,000,
are shown below: and freight-out was $140,000. Net sales last year for Pye Company were (E)
Merchandise inventory, Jan. 1, 1982 P 40,000 A. $12,720,000. C. $12,275,000.
Purchases 150,000 B. $12,450,000. D. $12,135,000. K, W & W
Purchase returns and allowances 5,000
Sales returns and allowances 750 Gross profit, purchases & ending inventory given
30. Hernandez Company had a gross profit of $240,000, total purchases of $280,000, and an B. 85,000 D. 265,000 CIA 0597 IV-15
ending inventory of $160,000 in its first year of operations as a retailer. Hernandez’s sales in
its first year must have been (E) . The total operating expenses of Travelodge Company for 2012 is 50% of cost of sales but
A. $360,000. C. $120,000. only 20% of sales. Finance costs are 5% of sales. The amount of purchases is 130% of costs
B. $440,000. D. $400,000. K, W & W of sales. Ending inventory is 25% greater than the beginning inventory. The profit for the
period after the tax of 30% is P2,450,000. How much were the beginning inventory? (D)
Common-size income statement A. 4,000,000 C. 6,000,000
55. A company has a 40% gross margin, general and administrative expenses of $50, interest B. 4,800,000 D. 5,000,000 HGT
expense of $20, and net income of $70 for the year just ended. If the corporate tax rate is
30%, the level of sales revenue for the year just ended was (E) Purchases
A. $170 C. $350 Cost of goods sold, inventory balances given
B. $255 D. $425 CIA 1194 IV-16 59. Haala Inc. is a merchandising company. Last month the company's cost of goods sold was
$68,000. The company's beginning merchandise inventory was $11,000 and its ending
56. A company has a 50% gross margin, general and administrative expenses of $50, interest merchandise inventory was $17,000. What was the total amount of the company's
expense of $20, and net income of $10 for the year just ended. If the corporate tax rate is merchandise purchases for the month? (M)
50%, the level of sales revenue for the year just ended was (E) A. $62,000 C. $74,000
A. $90 C. $150 B. $68,000 D. $96,000 G&N 12e
B. $135 D. $180 CIA 1194 IV-16
61. Haack Inc. is a merchandising company. Last month the company's cost of goods sold was
57. The profit of Imperial Company for the year ended December 31, 2012 was P480,000. $84,000. The company's beginning merchandise inventory was $20,000 and its ending
Percentage distribution of some of the items in profit or loss was as follows: merchandise inventory was $18,000. What was the total amount of the company's
 Selling expenses – 10% of sales merchandise purchases for the month? (E)
 Administrative expenses, excluding bad debts – 15% of sales ( also equal to 25% of cost A. $86,000 C. $84,000
of sales) B. $82,000 D. $122,000 G & N 9e
 Bad debts expenses – 3% of sales
What was the amount of Imperial Company’s sales during 2012? 81. Haaker Inc. is a merchandising company. Last month the company's cost of goods sold was
A. P1,920,000 C. P4,000,000 $87,000. The company's beginning merchandise inventory was $21,000 and its ending
B. P3,200,000 D. P4,800,000 R&E12 merchandise inventory was $18,000. What was the total amount of the company's
merchandise purchases for the month? (E)
Cost of Goods Sold A. $84,000 C. $90,000
Beginning Inventories B. $126,000 D. $87,000 G & N 10e
58. A retail enterprise maintains a markup of 25% based on cost. The enterprise has the
following information for 2001: 60. The following information was obtained from Smith Co.:
Purchases of merchandise 690,000 Sales $275,000
Freight-in on purchases 25,000 Beginning inventory 30,000
Sales 900,000 Ending inventory 18,000
Ending inventory 80,000 Smith's gross margin is 20%. What amount represents Smith purchases?
Beginning inventory was (E) A. $202,000 C. $220,000
A. 40,000 C. 110,000 B. $208,000 D. $232,000 AICPA R05
profit rate is expected to be 35%, compute the cost of the merchandise the owner should
79. Haagen Inc. is a merchandising company. Last month the company's cost of goods sold was expect to purchase during 2007.
$92,000. The company's beginning merchandise inventory was $14,000 and its ending A. $ 925,000. C. $1,025,000.
merchandise inventory was $16,000. What was the total amount of the company's B. $ 975,000. D. Some other amount.
merchandise purchases for the month? (E)
A. $92,000 C. $122,000 63. On July 1, the inventory of at Comfee Shoes was $50,000. Because of anticipated back-to-
B. $94,000 D. $90,000 G & N 10e school sales, the owner wants to have an inventory of $95,000 on hand at the beginning of
August. Net sales during July are expected to total $60,000, with a gross profit rate of 45%.
80. Haak Inc. is a merchandising company. Last month the company's cost of goods sold was During July, the company should purchase merchandise costing:
$62,000. The company's beginning merchandise inventory was $15,000 and its ending A. $57,500. C. $78,000.
merchandise inventory was $21,000. What was the total amount of the company's B. $128,000. D. Some other amount.
merchandise purchases for the month? (E)
A. $98,000 C. $68,000 64. At the beginning of 2007, England Dresses has an inventory of $140,000. However,
B. $62,000 D. $56,000 G & N 10e management wants to reduce the amount of inventory on hand to $80,000 at December 31. If
net sales for 2007 are forecast at $400,000 and the gross profit rate is expected to be 40%,
Cost of sales, inventory balances, freight-in, purchase discounts given compute the cost of the merchandise which management should expect to purchase during
33. All of the information required in the computation of cost of goods sold is presented below, 2007. (Hint: First compute the expected cost of goods sold.)
except for purchases, which must be what amount? A. $240,000. C. $320,000.
Purchase discounts $ 200 B. $180,000. D. Some other amount.
Inventory, December 31 1,500
Cost of goods sold 9,500 Cost of sales, ending inventory, average inventory, freight in, purchase returns & allowances, given
Purchases ? 3. The following are data of SIMPLE Corp. for 1986:
Inventory, January 1 1,500 Average merchandise inventory P170,000
Freight-in 500 Merchandise inventory, Dec. 31 165,000
A. $ 9,800 C. $ 8,700 Freight in 25,000
B. $10,200 D. $ 9,200 NB&J 11e Purchase return and allowances 35,000
Cost of sales 955,000
Sales & gross margin ratio, inventory balances given The gross purchases of SIMPLE Corp. in the year 1986 amounted to (M)
61. The following information was obtained from Smith Co.: A. P955,000 C. P965,000
Sales $275,000 B. P945,000 D. P975,000 RPCPA 1087
Beginning inventory 30,000
Ending inventory 18,000 Inventory balances, inventory turnover, freight-in, purchase returns & allowances given
Smith’s gross margin is 20%. What amount represents Smith purchases? 8. The following data pertain to Charter Co. for the year 1980:
A: $202,000 C: $220,000 Merchandise inventories
B: $208,000 D: $232,000 Wiley 2011 December 31, 1979 P50,000
December 31, 1980 60,000
62. At the beginning of 2007, Grand Hardware has an inventory of $300,000. Because sales Freight in 12,000
growth was strong during 2007, the owner wants to increase inventory on hand to $350,000 Purchase return and allowances 7,000
at December 31, 2007. If net sales for 2007 are expected to be $1,500,000, and the gross Inventory turnover rate 6
The gross purchase for the year amounted to (M) 27. The following information is available for the Brown Company for 2010:
A. P340,000 C. P330,000 Gross profit $ 30,000
B. P335,000 D. Answer not given RPCPA 0581 Net sales 500,000
Beginning inventory 220,000
Common-size income statement Ending inventory 40,000
6. The following information is provided by Eloisa Company for the current year. What was the amount of net purchases? (E)
Beginning inventory 400,000 A. $290,000 C. $180,000
Freight in
 300,000 B. $210,000 D. $150,000 NB&J 11e
Purchase returns 900,000
Ending inventory 500,000 66. An enterprise had the following account balances in the pre-closing trial balance:
Selling expenses 1,250,000 Opening inventory 100,000
Sales discount 250,000 Closing inventory 150,000
The cost of goods sold is six times the selling expenses. What is the amount of gross Purchases 400,000
purchases? Transportation-in 6,000
A. 6,500,000 C. 8,000,000 Purchase discounts 40,000
B. 6,700,000 D. 8,200,000 CPAR 1012 Purchase allowances 15,000
Returned purchases 5,000
Freight-in The enterprise had net purchases for the period of (E)
89. From the following information, determine the amount of freight-in. A. 340,000 C. 370,000
Beginning Inventory ................................... $20,000 B. 346,000 D. 376,000 CIA 1195
Purchases ............................................. 41,000
Purchase Returns and Allowances ....................... 3,000 Common-size income statement
Purchase Discounts .................................... 4,000 67. The total operating expenses of Travelodge Company for 2012 is 50% of cost of sales but
Freight-In ............................................ ? only 20% of sales. Finance costs are 5% of sales. The amount of purchases is 130% of costs
Cost of Goods Available for Sale ...................... 55,000 of sales. Ending inventory is 25% greater than the beginning inventory. The profit for the
Ending Inventory ...................................... ? period after the tax of 30% is P2,450,000. How much were the purchases during the year
Cost of Goods Sold .................................... 22,000 2012? (M)
A. $1,000 C. $3,000 A. P10,000,000 C. P4,000,000
B. $2,000 D. $4,000 S&S 19e B. P5,200,000 D. P1,200,000 R&E12

Net purchases Cost of Goods Available for Sale


65. The following information was obtained from Smith Co.: Net sales, gross margin, FG ending given
Sales
 $275,000 68. The following information is available for Sweden Company for its most recent year:
Beginning inventory 30,000 Net sales $3,600,000
Ending Inventory 18,000
 Freight-in 90,000
Smith's gross margin is 20%. What amount represents Smith purchases? Purchase discounts 50,000
A. $202,000 C. $220,000 Ending inventory 240,000
B. $208,000 D. $232,000 Becker 13 The gross margin is 40% of net sales. What is the cost of goods available for sale? (E)
A. $1,680,000 C. $2,400,000
B. $1,920,000 D. $2,440,000 AICPA 0582 93. The following information was obtained from the accounts of Foxx Company:
Beginning Inventory .................................. $20,000
69. The following information is available for Cooke Company for 2010: Purchases ............................................ 40,000
Net sales $1,800,000 Purchase Returns and Allowances ...................... 2,000
Freight-in 45,000 Purchase Discounts ................................... 4,000
Purchase discounts 25,000 Freight-In ........................................... 5,000
Ending inventory 120,000 Ending Inventory ..................................... 10,000
The gross margin is 40% of net sales. What is the cost of goods available for sale? Freight-Out .......................................... 6,000
A: $ 840,000 C: $1,200,000 Given this information, the cost of goods available for sale is
B: $ 960,000 D: $1,220,000 Wiley 2011 A. $59,000. C. $65,000.
B. $61,000. D. $69,000. S&S 19e
Gross sales, sales return & allow, gross margin, FG ending given
6. The following information was taken from the books of Batangas Co. for 1982: Beginning inventory, purchases, purchase returns, discount, freight in given
Gross sales P2,400,000 70. Outback Furriers started 2003 with $94,000 of merchandise inventory on hand. During 2003,
Inventory, December 31, 1982 180,000 $400,000 in merchandise was purchased on account with credit terms of 1/15 n/45. All
Purchases 1,800,000 discounts were taken. Purchases were all made f.o.b. shipping point. Outback paid freight
Freight-in 36,000 charges of $6,000. Merchandise with an invoice amount of $5,000 was returned for credit.
Sales returns and allowances 72,000 Cost of goods sold for the year was $380,000. Outback uses a perpetual inventory system.
Purchase discounts 22,000 Assuming Outback uses the gross method to record purchases, what is the cost of goods
Operating expenses 545,000 available for sale? (M2)
Gross margin on sales 30% A. $491,000. C. $489,000.
The cost of goods available for sale during 1982 was B. $495,000. D. $491,050. S, S & T
A. P1,800,000 C. P2,328,000
B. P1,809,600 D. None of these RPCPA 0583 71. Northwest Fur Co. started 2011 with $94,000 of merchandise inventory on hand. During
2011, $400,000 in merchandise was purchased on account with credit terms of 1/15, n/45. All
Decrease in inventory, purchases, purchase returns & allowances, purchase discounts, freight-in discounts were taken. Purchases were all made f.o.b. shipping point. Northwest paid freight
given charges of $7,500. Merchandise with an invoice amount of $5,000 was returned for credit.
88. Following are the account balances from Browne Company's income statement: Cost of goods sold for the year was $380,000. Northwest uses a perpetual inventory system.
Inventory, January 1, 2014 ............................ $35,000 Assuming Northwest uses the gross method to record purchases, what is the cost of goods
Purchases ............................................. 35,000 available for sale? (M2)
Purchase Returns and Allowances ....................... 2,000 A. $492,500. C. $490,500.
Purchase Discounts .................................... 4,000 B. $496,500. D. $492,550. S&S 6e
Freight-In ............................................ 5,000
Inventory, December 31, 2014 .......................... 10,000 72. Burnit Bakeries started 2003 with $62,000 of merchandise on hand. During 2003, $280,000 in
Freight-Out ........................................... 6,000 merchandise was purchased on account with credit terms of 2/10 n/30. All discounts were
Given this information, the cost of merchandise available for sale during 2014 is taken. Purchases were all made f.o.b. shipping point. Burnit paid freight charges of $9,000.
A. $59,000. C. $65,000. Merchandise with an invoice amount of $4,000 was returned for credit. Cost of goods sold for
B. $61,000. D. $69,000. S&S 19e the year was $316,000. Burnit uses a perpetual inventory system. What is cost of goods
available for sale, assuming Burnit uses the gross method? (M2)
A. $322,480. C. $336,000. available for sale?
B. $341,480. D. $347,000. S, S & T A. $600,000. C. $815,000.
B. $890,000. D. $860,000. KW&W 1e
73. Cinnamon Buns Co. (CBC) started 2011 with $52,000 of merchandise on hand. During 2011,
$280,000 in merchandise was purchased on account with credit terms of 2/10 n/30. All *. Tracy’s Sales reported the following information for 19x0:
discounts were taken. Purchases were all made f.o.b. shipping point. CBC paid freight Item Amount
charges of $9,000. Merchandise with an invoice amount of $4,000 was returned for credit. Purchase returns P100,000
Cost of goods sold for the year was $316,000. CBC uses a perpetual inventory system. Freight in 50,000
What is cost of goods available for sale, assuming CBC uses the gross method? (M2) Selling Expense 110,000
A. $312,480. C. $331,480. Ending Inventory 70,000
B. $326,000. D. $337,000. S&S 6e The cost of goods sold is equal to 500% of selling expenses. What is the cost of goods
available for sale? (M)
Common-size income statement A. P430,000 D. P450,000
40. The following information is available for Kerr Company for 2001: B. P620,000 E. P480,000
Freight-in $ 30,000 C. P600,000 RPCPA 0581
Purchase returns 75,000
Selling expenses 150,000 *. Records at Phil Orient Motors Co. disclose the following:
Ending inventory 260,000 For 1978
The cost of goods sold is equal to 300% of selling expenses. What is the cost of goods
Freight-in P 20,000
available for sale? (E)
Purchase returns 80,000
A. $450,000. C. $665,000.
Selling expenses 200,000
B. $740,000. D. $710,000. K, W & W
Ending inventory 90,000
24. The following information is given: The cost of goods sold is equal to 700% of selling expenses. What is the cost of goods
available for sale? (D)
Freight in P 2,000
A. P1,590,000 C. P1,490,000
Purchase returns 3,000
B. P1,500,000 D. P1,390,000 RPCPA 1078, 1080
Selling expenses 150,000
Ending inventory 80,000
75. The following information is given for Monte Carlo Company:
The cost of goods sold is equal to 800% of selling expenses. The cost of goods available for Freight-in P 2,000
sale is (M) Purchase returns 3,000
A. P1,200,000 C. P1,275,000 Selling expenses 150,000
B. P1,205,000 D. P1,280,000 RPCPA 1088 Ending inventory 80,000
The cost of goods sold is equal to 800% of selling expenses.
74. The following information is available for Naab Company for 2010: What is the cost of goods available for sale?
Freight-in $ 30,000 A. P1,200,000 C. P1,275,000
Purchase returns 75,000 B. P1,205, 000 D. P1,280,000 R&E12
Selling expenses 150,000
Ending inventory 260,000 Ending Inventory
The cost of goods sold is equal to 400% of selling expenses. What is the cost of goods Beg. Inventory, purchases, physical count
. Esmeralda Company uses the perpetual inventory system. On January 1, 2008, the balance B. P55,860 D. P67,032 RPCPA 0594
of the inventory account was P1,000,000 including goods costing P200,000 purchased in
transit, FOB shipping point that did not arrive until January 10, 2008. Purchases in 2008 Sales, markup, beginning inventory, purchases, purchase return
amounted to P12,000,000. The perpetual records showed an inventory balance of 76. Kesler, Inc. estimates the cost of its physical inventory at March 31 for use in an interim
P1,500,000 on December 31, 2008. A physical count taken also on the same date showed financial statement. The rate of markup on cost is 25%. The following account balances are
an inventory of P1,250,000. What amount should be reported as cost of goods sold for the available:
year 2008? (M1) Inventory, March 1 $220,000
A. 11,300,000 C. 11,550,000 Purchases 172,000
B. 11,500,000 D. 11,750,000 Siy Purchase returns 8,000
Sales during March 300,000
. Kelly Company uses the perpetual inventory system. On January 1, 2008, the balance of the The estimate of the cost of inventory at March 31 would be
inventory account was P2,000,000 including goods costing P500,000 purchased in transit, A. $84,000. C. $159,000.
FOB shipping point that did not arrive until January 5, 2008. Purchases in 2008 amounted to B. $144,000. D. $112,000. KW&W 1e
P15,000,000. The perpetual records showed an inventory balance of P2,500,000 on
December 31, 2008. A physical count taken also on the same date showed an inventory of Sales, Gross Margin, Beginning Inventory, Purchases
P2,650,000. What amount should be reported as cost of goods sold for the year 2008? 77. The following amounts relate to an enterprise:
A. 14,150,000 C. 14,500,000 Beginning inventory 50,000
B. 14,350,000 D. 14,650,000 Siy Purchases 170,000
Sales 180,000
Cost of Goods Available for Sale, Cost of Goods Sold Gross margin 60,000
90. From the following information, determine the amount of ending inventory. The amount of ending inventory is (E)
Beginning Inventory ................................... $20,000 A. 60,000 C. 120,000
Purchases ............................................. 41,000 B. 100,000 D. 160,000 CIA 0597
Purchase Returns and Allowances ....................... 3,000
Purchase Discounts .................................... 4,000 58. Delta Merchandising, Inc., has provided the following information for the year just ended:
Freight-In ............................................ ? Net sales $128,500
Cost of Goods Available for Sale ...................... 55,000 Beginning inventory 24,000
Ending Inventory ...................................... ? Purchases 80,000
Cost of Goods Sold .................................... 22,000 Gross margin 38,550
A. $22,000 C. $32,000 The ending inventory for the company at year end was: (E)
B. $23,000 D. $33,000 S&S 19e A. $65,450. C. $14,050.
B. $24,500. D. $9,950. G & N 9e
Purchases, Discount, Ending inventory as % of purchases
22. The T Corporation made purchases of P114,000 during the year on terms of 2/10, n/30. The 39. The following information is available for the Underwater Company for the three months
company took advantage of the discount on 60% of the purchases. It paid for the remainder ended March 31 of this year:
of the account after the discount period had expired. The company uses the periodic Inventory, January 1 .................................. $ 450,000
inventory system. Half of the purchases are still in the inventory at the end of the year. Purchases ............................................. 1,700,000
The cost of ending inventory is (E) Freight-in ............................................ 100,000
A. P56,772 C. P56,316 Sales ................................................. 2,400,000
The gross margin was estimated to be 25 percent of sales. What is the estimated inventory B. P59,000 D. None of these. RPCPA 1077
balance at March 31?
A. $350,000 C. $562,500 21. Montana Company is a wholesale electronics distributor. On December 31, 2014, it prepared
B. $450,000 D. $600,000 S&S 19e the following partial income statement:
Gross sales ............................... $800,400
Sales, gross margin ratio, cost of goods available for sale Sales discounts ........................... 400
78. For 2010, cost of goods available for sale for Tate Corporation was $900,000. The gross Net sales ................................. $800,000
profit rate was 20%. Sales for the year were $800,000. What was the amount of the ending Cost of goods sold:
inventory? Beginning inventory ..................... $300,000
A. $0. C. $180,000. Net purchases ........................... 300,000
B. $260,000. D. $160,000. KW&W 1e Given this information, if Montana Company's gross margin is 30 percent of net sales, what is
the correct ending inventory balance? (M)
Sales, gross margin ratio, beginning inventory, purchases given A. $40,000 C. $360,000
*. Beginning inventory was P35,000, purchases were P146,000, and sales totaled P240,000. B. $240,000 D. $600,000 S&S 19e
With normal gross margin of 35%, how much is ending inventory. (E)
A. P181,000 C. P35,000 Sales, gross margin ratio, beginning inventory, purchases, freight-in given
B. P25,000 D. P97,000 RPCPA 0598 80. The following information is available for the Silver Company for the 3 months ended March
31, 2011.
3. The merchandise inventory of CD Ltd. On January 1, 1988 was P450,000. During 1988, the Merchandise inventory, January 1, 2011 $ 900,000
company has recorded sales of P1,500,000 and merchandise purchases of P1,100,000. The Purchases 3,400,000
gross profit (margin) on these sales was 30%. Freight-in 200,000
What is the merchandise inventory as at December 31, 1988? Sales 4,800,000
A. P500,000 C. P1,100,000 The gross margin recorded was 25% of sales. What should be the merchandise inventory at
B. P650,000 D. Answer not given. RPCPA 1089 March 31, 2011?
A: $ 700,000 C: $1,125,000
79. On January 1, 2010, the merchandise inventory of Glaus, Inc. was $800,000. During 2010 B: $ 900,000 D: $1,200,000 Wiley 2011
Glaus purchased $1,600,000 of merchandise and recorded sales of $2,000,000. The gross
profit rate on these sales was 25%. What is the merchandise inventory of Glaus at December Beginning inventory, purchases, freight-in, discounts, cost of sales given
31, 2010? 81. Outback Furriers started 2003 with $94,000 of merchandise inventory on hand. During 2003,
A. $400,000. C. $900,000. $400,000 in merchandise was purchased on account with credit terms of 1/15 n/45. All
B. $500,000. D. $1,500,000. KW&W 1e discounts were taken. Purchases were all made f.o.b. shipping point. Outback paid freight
charges of $6,000. Merchandise with an invoice amount of $5,000 was returned for credit.
7. From the information below for the period July 1 to September 30, compute the estimated Cost of goods sold for the year was $380,000. Outback uses a perpetual inventory system.
inventory at September 30: What is ending inventory assuming Outback uses the gross method to record purchases? (M)
Sales, net, for the period 440,000 A. $110,990. C. $115,000.
Gross profit (margin) on sales 35% B. $111,050. D. $118,800. S, S & T
Inventory, July 1 P 45,000
Purchases, net, for the period 300,000 82. Burnit Bakeries started 2003 with $62,000 of merchandise on hand. During 2003, $280,000 in
A. P95,000 C. P54,000 merchandise was purchased on account with credit terms of 2/10 n/30. All discounts were
taken. Purchases were all made f.o.b. shipping point. Burnit paid freight charges of $9,000. What is the inventory on December 31, 2014? (M2)
Merchandise with an invoice amount of $4,000 was returned for credit. Cost of goods sold for A. 2,025,000 C. 2,505,000
the year was $316,000. Burnit uses a perpetual inventory system. Assuming Burnit uses the B. 2,370,000 D. 3,285,000 FA © 2014
gross method to record purchases, ending inventory would be: (M)
A. $16,480. C. $31,000. Common size financial statements
B. $25,400. D. $25,480. S, S & T . The total operating expenses of Travelodge Company for 2012 is 50% of cost of sales but
only 20% of sales. Finance costs are 5% of sales. The amount of purchases is 130% of costs
Beg. inventory, purchases, purchase returns, purchase discounts, freight charges, cost of goods of sales. Ending inventory is 25% greater than the beginning inventory. The profit for the
sold period after the tax of 30% is P2,450,000. How much were the ending inventory? (D)
83. Northwest Fur Co. started 2011 with $94,000 of merchandise inventory on hand. During A. 4,000,000 C. 5,000,000
2011, $400,000 in merchandise was purchased on account with credit terms of 1/15, n/45. All B. 4,800,000 D. 6,000,000 HGT
discounts were taken. Purchases were all made f.o.b. shipping point. Northwest paid freight
charges of $7,500. Merchandise with an invoice amount of $5,000 was returned for credit. Shipping cost in ending inventory
Cost of goods sold for the year was $380,000. Northwest uses a perpetual inventory system. 86. Seafood Trading Co. commenced operations during the year as a large importer and exporter
What is ending inventory assuming Northwest uses the gross method to record purchases? of seafood. The imports were all from one country overseas. The export sales were
(D) conducted as drop shipments and were merely transshipped at Seattle. Seafood Trading
A. $112,490. C. $116,500. reported the following data:
B. $112,550. D. $120,300. S&S 6e Purchases during the year $12.0 million
Shipping costs from overseas 1.5 million
84. Cinnamon Buns Co. (CBC) started 2011 with $52,000 of merchandise on hand. During 2011, Shipping costs to export customers 1.0 million
$280,000 in merchandise was purchased on account with credit terms of 2/10 n/30. All Inventory at year-end 3.0 million
discounts were taken. Purchases were all made f.o.b. shipping point. CBC paid freight What amount of shipping costs should be included in Seafood Trading’s year-end inventory
charges of $9,000. Merchandise with an invoice amount of $4,000 was returned for credit. valuation?
Cost of goods sold for the year was $316,000. CBC uses a perpetual inventory system. A: $0 C: $375,000
Assuming CBC uses the gross method to record purchases, ending inventory would be: (D) B: $250,000 D: $625,000 AICPA R08
A. $6,480. C. $15,480.
B. $15,400. D. $21,000. S&S 6e Cost of Goods Sold
Purchases, inventory balances given
Sales, Beg. inventory, purchases, freight-in, purchase discounts, returns & allowances 87. Given a periodic inventory system, beginning inventory of $30,000, ending inventory of
85. Fairy Company provided the following information: $34,000, and purchases of $450,000, what is the net debit to cost of goods sold?
2013 2014 A. $450,000 C. $34,000
Sales 7,500,000 4,500,000 B. $30,000 D. $446,000
Beginning inventory 1,260,000
Purchases 6,450,000 3,180,000 139. At the beginning of September, 2008, RFI Company reported Merchandise Inventory of
Freight in 350,000 220,000 $4,000. During the month, the company made purchases of $7,800. At September 31, 2008,
Purchase discounts 90,000 45,000 a physical count of inventory reported $3,200 on hand. Cost of goods sold for the month is
Purchase returns 120,000 40,000 A. $600. C. $8,600.
Purchase allowances 20,000 15,000 B. $7,800. D. $11,800.
Ending inventory 2,355,000 ?
60. Gabel Inc. is a merchandising company. Last month the company's merchandise purchases A. $720 C. $360
totaled $63,000. The company's beginning merchandise inventory was $13,000 and its B. $550 D. $140
ending merchandise inventory was $15,000. What was the company's cost of goods sold for
the month? (E) 89. Michael uses its periodic inventory system and the following information is available:
A. $91,000 C. $65,000 Sales $43,400
B. $63,000 D. $61,000 G & N 9e Inventory – Beginning 11,200
Inventory – Ending 9,800
78. Gabriel Inc. is a merchandising company. Last month the company's merchandise purchases Purchases 32,200
totaled $70,000. The company's beginning merchandise inventory was $15,000 and its Calculate the cost of goods sold:
ending merchandise inventory was $22,000. What was the company's cost of goods sold for A. $ 9,800. C. $32,200.
the month? (E) B. $33,600. D. $43,400.
A. $63,000 C. $107,000
B. $77,000 D. $70,000 G & N 10e 90. Rigmarole Company used a perpetual inventory system. At the end of 2013, the balance in
the inventory account was P360,000 which included P30,000 of goods purchased FOB
77. Gabor Inc. is a merchandising company. Last month the company's merchandise purchases shipping point that did not arrive until 2014. Purchases in 2014 were P3,000,000. The
totaled $89,000. The company's beginning merchandise inventory was $13,000 and its perpetual inventory records showed an ending inventory of P420,000 for 2014. A physical
ending merchandise inventory was $16,000. What was the company's cost of goods sold for count of the goods on hand at the end of 2014 showed an inventory of P380,000. Inventory
the month? (E) shortages are included in cost of goods sold. What amount should be reported for cost of
A. $89,000 C. $118,000 goods sold in 2014?
B. $86,000 D. $92,000 G & N 10e A. 2,940,000 C. 3,000,000
B. 2,980,000 D. 3,010,000 FA © 2014
76. Gable Inc. is a merchandising company. Last month the company's merchandise purchases
totaled $86,000. The company's beginning merchandise inventory was $15,000 and its Purchase, inventory balances, drawing given
ending merchandise inventory was $11,000. What was the company's cost of goods sold for 22. Assume that A had an opening inventory of P10,000; he purchased goods for resale during
the month? (E) the year at a total cost of P16,000; the year’s total sales were P30,000; and the closing
A. $86,000 C. $82,000 inventory was P6,000. The cost of merchandise withdrawn by A for his personal use was
B. $112,000 D. $90,000 G & N 10e P2,000. The cost of goods sold is computed at
A. P12,000 C. P16,000
88. Gabrio Inc. is a merchandising company. Last month the company's merchandise purchases B. P14,000 D. P18,000 RPCPA 0578
totaled $87,000. The company's beginning merchandise inventory was $19,000 and its
ending merchandise inventory was $11,000. What was the company's cost of goods sold for Purchases, freight-in, inventory balances, given
the month? (E) 91. A company starts the year with inventory costing $120,000. During the year, inventory costing
A. $79,000 C. $95,000 $340,000 was acquired. Freight costs to get the inventory to the company amounted to
B. $87,000 D. $117,000 G&N 12e $19,000. Freight costs to get the inventory to customers after the goods were acquired
totaled $17,000. At the end of the year, inventory costing $165,000 was still on hand. What
7. A company, using the periodic inventory system, has merchandise inventory costing $140 on was the cost of good sold for the period?
hand at the beginning of the period. During the period, merchandise costing $400 is A $295,000 C $314,000
purchased. At year-end, merchandise inventory costing $180 is on hand. The cost of B $312,000 D $331,000
merchandise sold for the year is (E)
92. Glen Company has the following data pertaining to the year ended December 31: What amount should Azur report as cost of goods sold for the year? (E)
Purchases $450,000 A. $102,800 C. $123,360
Beginning inventory 170,000 B. $118,220 D. $128,500 AICPA R99
Ending inventory 210,000
Freight-in 50,000 Purchase, freight-in, purchase return, inventory balances given
Freight-out 75,000 32. From the following information, compute cost of goods sold.
How much is the cost of goods sold for the year? (E) Purchase returns $ 200
A. $385,000 C. $485,000 Inventory, December 31 1,500
B. $460,000 D. $535,000 AICPA 0586 I-44 Freight-in 100
Inventory, January 1 1,800
93. Glen Company has the following data pertaining to the year ended December 31, 2004: Purchases 5,000
Purchases 4,500,000 A. $5,300 C. $5,100
Beginning inventory 1,700,000 B. $5,200 D. $5,000 NB&J 11e
Ending inventory 2,100,000
Freight in 500,000 Purchases, freight-in, purchase returns & allowances, discounts, inventory balances given
Freight out 750,000 96. An enterprise had the following account balances in the pre-closing trial balance:
How much is the cost of goods sold for 2004? (E) Opening inventory 100,000
A. 3,850,000 C. 4,850,000 Closing inventory 150,000
B. 4,600,000 D. 5,400,000 AICPA Purchases 400,000
Transportation-in 6,000
Purchases, purchase discount, inventory balances given Purchase discounts 40,000
94. The following information is available from Carron Company's 2014 accounting records: Purchase allowances 15,000
Purchases ............................................ $530,000 Returned purchases 5,000
Purchase discounts ................................... 10,000 If net purchases for the enterprise equal 500,000 for the period, cost of goods sold is (E)
Beginning inventory .................................. 160,000 A. 250,000 C. 550,000
Ending inventory ..................................... 215,000 B. 450,000 D. 750,000 CIA 1195 IV-5 & 6
Freight-out .......................................... 40,000
Carron's 2014 cost of goods sold is 91. The following information was obtained from the accounts of Marion Company:
A. $465,000. C. $505,000. Inventory, January 1 .................................. $30,000
B. $475,000. D. $585,000. S&S 19e Purchases ............................................. 45,000
Purchase Returns and Allowances ....................... 5,000
Purchase, freight-in, purchase discount, inventory balances given Purchase Discounts .................................... 4,000
95. The following information pertained to Azur Co. for the year: Freight-In ............................................ 5,000
Purchases $102,800 Inventory, December 31 ................................ 20,000
Purchase discounts 10,280 Freight-Out ........................................... 6,000
Freight-in 15,420 Given this information, the cost of goods sold during the year is
Freight-out 5,140 A. $41,000. C. $51,000.
Beginning inventory 30,840 B. $46,000. D. $61,000. S&S 19e
Ending inventory 20,560
87. Following are the account balances from Canarsie Company's income statement: Inventory, December 31 1,500,000
Inventory, January 1, 2014 ............................ $30,000 What amount should Malabon report as cost of goods sold?
Purchases ............................................. 40,000 A. 21,500,000 C. 23,000,000
Purchase Returns and Allowances ....................... 5,000 B. 23,500,000 D. 25,000,000 CPAR 4118
Purchase Discounts .................................... 4,000
Freight-In ............................................ 5,000 Purchase Cutoff, Inventory Shortage
Inventory, December 31, 2014 .......................... 15,000 99. BR Company uses a perpetual inventory system and follows GAAP in preparing its external
Freight-Out ........................................... 6,000 financial statements. At the end of 2003, the balance in the inventory account was P360,000
Given this information, the cost of goods sold during 2014 is and P30,000 of those goods included in ending inventory were purchased FOB shipping point
A. $46,000. C. $56,000. and did not arrive until 2004. Purchases in 2004 were P3,000,000. The perpetual inventory
B. $51,000. D. $66,000. S&S 19e records showed an ending inventory of P420,000 for 2004. A physical count of the goods on
hand at the end of 2004 showed an inventory of P380,000. Inventory shortages are included
Purchases, write-off, inventory balances given in cost of goods sold. What should the company report on its 2004 income statement for cost
97. The following information pertains to Hague Corp.’s 2001 cost of goods sold: of goods sold? (M)
Inventory, 12/31/00 $180,000 A. 2,940,000 C. 3,000,000
2001 purchases 248,000 B. 2,980,000 D. 3,010,000 CGAC
2001 write-off of obsolete inventory 68,000
Inventory, 12/31/01 60,000 Purchases, freight in, consignment
The inventory written off became obsolete because of an unexpected and unusual 100. During the current year, Kam Co. began offering its goods to selected retailers on a
technological advance by a competitor. In its 2001 income statement, what amount should consignment basis. The following information was derived from Kam's current year
Hague report as cost of goods sold? (E) accounting records:
A. $436,000 C. $300,000 Beginning inventory $122,000
B. $368,000 D. $248,000 AICPA 0591 I-48 Purchases
 540,000
Freight in
 10,000
98. The following information pertains to Deal Corp.’s 2003 cost of goods sold: Transportation to consignees 5,000
Inventory, 12/31/02 $ 90,000 Freight out 35,000
2003 purchases 124,000 Ending inventory (held by Kam ) 145,000
2003 write-off of obsolete inventory 34,000 Ending inventory (held by consignees) 20,000
Inventory, 12/31/03 30,000 In year-end income statement, what amount should Kam report as cost of goods sold?
The inventory written off became obsolete due to an unexpected and unusual technological A. $507,000 C. $527,000
advance by a competitor. In its 2003 income statement, what amount should Deal report as B. $512,000 D. $547,000 Becker 13
cost of goods sold?
A. $218,000 C. $150,000 Net Purchases & Cost of Goods Sold
B. $184,000 D. $124,000 AICPA 0593 Questions 112 and 113 are based on the following information. CIA 1195
An enterprise had the following account balances in the pre-closing trial balance:
1. The following information pertains to Malabon Company’s 2003 cost of goods sold. Opening inventory 100,000
Inventory, January 1 5,000,000 Closing inventory 150,000
Purchases 20,000,000 Purchases 400,000
Writeoff of obsolete inventory 2,000,000 Transportation-in 6,000
Purchase discounts 40,000 A. $190,000 C. $160,000
Purchase allowances 15,000 B. $180,000 D. $150,000 AICPA 1189
Returned purchases 5,000
106. For Mortenson Company, the following information is available:
101. The enterprise had net purchases for the period of Cost of goods sold $ 60,000
A. 340,000 C. 370,000 Sales discounts 2,000
B. 346,000 D. 376,000 Income tax expense 6,000
Operating expenses 23,000
102. If net purchases for the enterprise equal 500,000 for the period, cost of goods sold is Sales 100,000
A. 250,000 C. 550,000 In Mortenson’s income statement, gross profit KW&W 1e
B. 450,000 D. 750,000 A. should not be reported. C. should be reported at $38,000.
B. should be reported at $9,000. D. should be reported at $40,000.
Gross Margin
103. Gamma Ray Corp. has annual sales totaling $650,000 and an average gross profit of 20% of 107. For Rondelli Company, the following information is available:
cost. What is the dollar amount of the gross profit? Cost of goods sold $ 90,000
A. $130,000. C. $108,333. Sales returns and allowances 4,000
B. $97,500. D. $162,500. KW&W 1e Income tax expense 9,000
Operating expenses 35,000
104. The following data appeared in the accounting records of a retail store for the year ended Sales 150,000
December 31, 2010: In Rondelli's income statement, gross profit KW&W 1e
Sales $150,000 A. should not be reported. C. should be reported at $56,000.
Purchases 70,000 B. should be reported at $12,000. D. should be reported at $60,000.
Inventories:
January 1 35,000 108. A company has the following items on its year-end trial balance:
December 31 50,000 Net sales
 $500,000
Sales commissions 5,000 Common stock 100,000
How much was the gross margin? Insurance expense 75,000
A: $65,000 C: $90,000 Wages
 50,000
B: $75,000 D: $95,000 Wiley 2011 Cost of goods sold 100,000
Cash
 40,000
105. The following information appeared in the accounting records of a retail store for the year Accounts payable
 25,000
ended December 31, 1988: Interest payable 20,000
Sales $300,000 What is the company's gross profit?
Purchases 140,000 A. $230,000 C. $400,000
Inventories B. $275,000 D. $500,000 Becker 13
January 1 70,000
December 31 100,000 Common-size income statement
Sales commissions 10,000 109. After the initial year of operations, the Wynn Company had the following data in its operating
The gross margin was: results:
Net profit is P750, 000. Selling expenses are 12.5% of sales and also equal to 25% of cost of 24. Knitness Menswear, Inc. maintains a markup of 60 percent based on cost. The company's
goods sold. Administrative expenses and other expenses are 17.5% and 5% of sales, selling and administrative expenses average 30 percent of sales. Annual sales were
respectively. $1,440,000. Petersen's cost of goods sold and operating profit for the year are
What is Wynn’s gross profit for the year? S&S 19e A. B. C. D.
A. P5,000,000 C. P1,750,000 Cost of goods sold $864,000 $864,000 $900,000 $900,000
B. P2,500,00 D. P750,000 R&E12 Operating profit $144,000 $432,000 $108,000 $432,000

Gross profit rate Net Income


110. Illusive Company provided the following data for the current year: 113. In its first year of operations Best Corp. had income before tax of $500,000. Best made
Sales 6,200,000 income tax payments totaling $210,000 during the year and has an income tax rate of 40%.
Sales return 200,000 What was Best's net income for the year? (E)
Inventory, January 1 1,000,000 A. $290,000. C. $300,000.
Purchases 5,500,000 B. $294,000. D. $306,000. SS&T
Freight in 250,000
Purchase return 100,000 114. Zeno Company maintains a markup of 60% based on cost. The entity's distribution and
Purchase allowance 30,000 administrative expenses average 30% of sales. Sales amounted to P9,600,000 for current
Purchase discount 20,000 year. What is the net income for the current year?
Inventory, December 31 2,100,000 A. 720,000 C. 2,880,000
What is the gross profit rate on cost for the current year? B. 960,000 D. 3,600,000 FA © 2014
A. 25 percent C. 75 percent
B. 33 1/3 percent D. 66 2/3 percent FA © 2014 115. Ronalyn Company reported that the financial records were destroyed by fire at the end of the
current year. However, certain statistical data related to the income statement are available.
Mark-up & Margin Interest expense 20,000
13. A markup of 25 percent on cost is equivalent to what markup on selling price? (E) Cost of goods sold 2,000,000
A. 15 percent C. 25 percent Sales discount 100,000
B. 20 percent D. 33 percent S,S&S, Valix 2012 The beginning inventory was P400,000 and decreased 20% during the year. Administrative
expenses are 25% of cost of goods sold but only 10% of gross sales. Four-fifths of the operating
111. A markup of 40% on cost is equivalent to what margin on selling price? expenses relate to sale activities. Ignoring income tax, what is the net income for the current
A. 29% C. 60% year?
B. 40% D. 71% KW&W 1e A. 330,000 C. 400,000
B. 380,000 D. 480,000 FA © 2014
112. The sales price for a product provides a gross profit of 25% of sales price. What is the gross
profit as a percentage of cost? COMPREHENSIVE
A. 25%. Questions 1 and 2 are based on the following information. RPCPA 1089
B. 20%. The following information is made available from the records of AB & Co. for 1988:
C. 33%. Beginning inventory P 80,000
D. Not enough information is provided to determine. KW&W 1e Freight in 25,000
Purchase returns 80,000
Cost of goods sold & operating expenses Ending inventory 100,000
Selling expenses 250,000 debt was issued on July 1 of the current year and originally had 5 years to maturity.
Sales discounts 15,000  The fixed assets have a 10-year estimated useful life and were 1 year old at the start of the
The Cost of Goods Sold is six (6) times the selling expenses. current year. Straight-line depreciation is used by the company.
Cash $80,000
2. What is the total purchases? Accounts receivable 100,000
A. P1,520,000 C. P1,600,000 Inventory 230,000
B. P1,575,000 D. Answer not given. Gross fixed assets 600,000
Accumulated depreciation 60,000
1. What is the cost of goods sold available for sale? Accounts payable 200,000
A. P1,500,000 C. P1,600,000 Long-term debt 1,000,000
B. P1,550,000 D. Answer not given. Common stock 2,000,000
Retained earnings (Jan. 1) 500,000
Questions 38 thru 40 are based on the following information. R&E12 Sales revenue 750,000
The 2012 operations of Ballys Company resulted in the following: Purchases 530,000
 The cost of goods sold amounted to P350,000. Administrative expenses 200,000
 The beginning inventory is P500,000 greater than ending inventory, the latter being equivalent
to 20% of purchases during the period. 119. The company uses straight-line depreciation for financial reporting purposes, but uses
 Gross profit of the company is 30% of net sales. accelerated depreciation for tax purposes. Which of the following account balances would be
 Total operating expenses amounted to 60% of the gross profit while sales returns amounted to lower in the financial statements used for tax purposes than it would be in the general
2% of net sales. purpose financial statements?
 The company is subject to income tax rate of 30%. A. Accumulated depreciation. C. Gross fixed assets.
B. Retained earnings. D. Accounts receivable. CIA 0594 IV-1
116. What was the company’s net sales for the period?
A. P3, 500, 000 C. P5, 000,000 120. On the year-end financial statements, the company will report cost of goods sold of
B. P4, 100, 000 D. P5, 100.000 A. $440,000 C. $620,000
B. $530,000 D. $670,000 CIA 0594 IV-2
117. What was the amount of profit after income tax?
A. P180, 000 C. P600, 000 121. The company will report year-end total assets of
B. P420, 000 D. P900, 000 A. $800,000 C. $950,000
B. $890,000 D. $1,010,000 CIA 0594 IV-5
118. What were the total purchases for the period?
A. P 600,000 C. P3, 500, 000 122. Which adjusting entry should be used at year-end to account for interest expense on the
B. P 3, 000, 000 D. P4, 000. 000 long-term debt? CIA 0594 IV-4
A. Interest expense $100,000
Questions 71 through 76 are based on the following information. Interest payable $100,000
A company's pre-closing trial balance and other pertinent information at December 31 are as B. Interest expense $50,000
follows. Cash $50,000
 The balance of opening inventory was $140,000. C. Interest payable $100,000
 The long-term debt pays interest at a rate of 10% per annum, payable every 12 months. The Interest expense $100,000
D. Interest expense $50,000
Interest payable $50,000 4. Choice "C" is correct. $2, 944
Amount net of trade discount (5,000 x 70% x 80%) 2,800
123. Assume that the company reports cost of goods sold of $200,000 and interest expense of Cash discount (2,800 x 2%) (56)
$10,000 for the current period. Also assume a 50% tax rate on corporate earnings. The final Loan of delivery cost 200
closing entry required to ensure that current earnings are incorporated into year-end retained Expected remittance 2,944
earnings is CIA 0594 IV-6
A. Income summary $140,000 5. Answer is (C).
Retained earnings $140,000 Invoice price (1,000,000 x .80 x .90) 720,000
B. Retained earnings $280,000 Cash discount (5% x 720,000) (36,000)
Income summary $280,000 Net amount 684,000
C. Income summary $240,000 Freight charge 50,000
Retained earnings $240,000 Total remittance 794,000
D. Retained earnings $240,000
Income summary $240,000 6. $80,000 ($100,000 x 80%)

7. Choice "b" is correct. $14,400. ($20,000 x 80% x 90%) = $14,400)


ANSWER EXPLANATIONS
8. b $50,000 × .8 × .9 = $36,000.
1. B List price 5,000,000
Trade discount – 20% (1,000,000) 9. $21,600 ($30,000 x 80% x 90%)
Balance 4,000,000
Trade discount – 5% (200,000) 10. REQUIRED: The amount to be recorded as cost of inventory subject to trade discounts.
Invoice price 3,800,000 DISCUSSION: (B) When inventory is subject to cash discounts, the purchases may be
VAT – 10% 380,000 reflected either net of these discounts or at the gross prices. However, purchases should
Accounts receivable 4,180,000 always be recorded net of trade discounts. A chain discount is the application of more than
one trade discount to a list price. Chain discounts should be applied in steps as indicated
2. C below.
List price 5,000,000 List price $20,000
Trade discounts - 20% (1,000,000) 20% discount (4,000)
Balance 4,000,000 $16,000
Trade discounts - 5% ( 200,000) 10% discount (1,600)
Invoice price 3,800,000 Cost of merchandise $14,400
Vat - 10% 380,000 Answer (A) is incorrect because $14,000 applies both discounts to the retail price. Answer (C)
Accounts receivable 4,180,000 is incorrect because $15,600 assumes the 10% discount is applied to the 20% discount.
Answer (D) is incorrect because $20,000 is the list price, and it fails to reflect the discounts.
3. Answer is (B). 450,000 x 1.4 = 630,000
630,000 – 585,000 = 45,000 11. P109,012,50 (P150,000 x 85% x 90% x 95%)
12. Answer is (D). (D) is incorrect because no inventory should be included in the 2001 financial statements, and
Invoice price (900,000 x .80 x .90) 648,000 freight-in should be recorded in 2002.

13 . List price 600,000 18. (d) When the shipping terms are FOB destination, the seller bears all costs of transporting the
Trade discounts (20% x 600,000) (120,000) goods to the buyer. Therefore, the seller is responsible for the payment of packaging costs
Balance 480,000 ($1,000), shipping costs ($1,500), and the special handling charges ($2,000). The only amount
(10% x 480,000) ( 48,000) to be included as the buyer’s cost of the inventory purchased is the purchase price ($50,000).
Invoice price 432,000
Freight charge 15,000 19. Answer A is correct. The costs to be charged to merchandise purchases should include those
Total cost of purchase 447,000 costs necessary to prepare the merchandise for sale. Salesmen's commissions are a selling
expense and not related to the acquisition of the merchandise. These costs are expensed in
Purchases are normally recorded at gross. Thus, the cash discount is ignored. the period incurred. The interest is a financing expense and is also expensed in the period
incurred. Thus, only the $500,000 should be included in the cost of the merchandise
14. Answer is (D). 90,000 x .80 x ..90 = 64,800; 64,800 + 5,000 = 69,800 purchases.

15. Answer is (D). 20. REQUIRED: The amount of cost for goods included in inventory.
Cost of purchases 5,000,000 DISCUSSION: (D) FOB destination means that title passes upon delivery at the destination,
Import duties the seller bears the risk of loss, and the seller is responsible for the expense of delivering the
400,000
goods to the designated point. Consequently, the packaging, shipping, and handling costs are
Freight and insurance 1,000,000 not included in the inventory. The amount that should be included is therefore the purchase
Other handling costs 100,000 price of $50,000.
Brokerage commission 200,000 Answer (A) is incorrect because the packaging, shipping, and handling costs should not be
Total cost of purchases 6,700,000 included. Answer (B) is incorrect because the shipping and handling costs should not be
included. Answer (C) is incorrect because the handling costs should not be included.
16. Answer (D) is correct. The shipping term indicates that title and risk of loss passed to the
buyer at the shipping point. Hence, the 2001 ending inventory should include the 2,000 cost of 21 . A Cost of inventory 5,000,000
this purchase. Also, the buyer is responsible for 50 of freight regardless of which party initially
paid. The seller bears the expense of delivery to the shipping point, not the destination. 22. (c) Inventoriable costs include all costs necessary to prepare goods for sale. For a
Answer (A) is incorrect because inventory and freight-in should be 2,000 and 50, respectively. merchandising concern, these include the purchase price of the goods, freight-in, insurance,
Answer (B) is incorrect because inventory and freight-in should be 2,000 and 50, respectively. warehousing, and any costs necessary to get the goods to the point of sale. Abnormal freight
Answer (C) is incorrect because inventory should be 2,000. and handling should be charged to expense of the period. Therefore, the normal costs for
inventory are $5,500 ($3,000 + $2,000 + $500) and the abnormal freight of $1,200 is charged
17. Answer (A) is correct. Title and risk of loss passed to the buyer at the destination, and the to current expense of the period. ·
seller incurred the expense of delivery to that point. The goods did not arrive until after year-
end, so they should not be included in 2001 inventory. Freight-in should also not be recorded 23. c $300,000 + $8,000 – $2,000 = $306,000.
until 2002.
Answer (B) is incorrect because no inventory should be included in the 2001 financial 24. REQUIRED: The amount of inventoriable cost for the year.
statements. Answer (C) is incorrect because freight-in should be recorded in 2002. Answer
DISCUSSION: (C) Inventoriable cost is the sum of the applicable expenditures and charges
directly or indirectly incurred in bringing all items of inventory to their existing condition and 29. $980 ($1,000 x 98%)
location. Thus, inventoriable cost includes the $400,000 cost of the merchandise purchased,
plus the $10,000 of freight-in, minus the $2,000 of purchase returns. Freight-out is not a cost 30. P5,697.72 (P8,000 x 85% x 90% x 95% x 98%)
incurred in bringing the inventory to a salable condition. Consequently, the inventoriable cost
for Fenn was $408,000 ($400,000 + $10,000 – $2,000). 31. P106,832.25 (P109,012.50 x 98%)
Answer (A) is incorrect because $400,000 excludes freight-in and purchase returns. Answer
(B) is incorrect because $403,000 excludes freight-in. Answer (D) is incorrect because 32. Answer is (A). 109,012.50 x .98 = 106,832.25
$413,000 includes freight-out.
33. (c) Purchases are always recorded net of trade discounts. When more than one trade
25. REQUIRED: The amount of inventoriable cost. discount is applied to a list . price, it is called a chain discount. Chain discounts are applied
DISCUSSION: (C) Inventoriable cost is the sum of the applicable expenditures and charges in steps; each discount applies to the previously discounted price. The cost, net of trade
directly or indirectly incurred in bringing all items of inventory to their existing condition and discounts, is $2,800 [$5,000 – (30% x $5,000) = $3,500; and $3,500 – (20% x $3,500) =
location. Thus, inventoriable cost includes the $800,000 cost o the merchandise purchased, $2,800]. Payment was made within the discount period, so the net purchase price is $2,744
plus the $20,000 of freight-in, minus the $4,000 of purchase returns. Freight-out is not a cost [$2,800 – (2% x $2,800)]. The remittance from Burr would also include reimbursement of the
incurred in bringing the inventory to a salable condition. The inventoriable cost for Atlas during $200 of delivery costs. Since the terms were FOB shipping point, Burr is responsible for
2001 is $816,000 ($800,000 + $20,000 - $4,000) paying this amount, and must reimburse Pitt, who prepaid the freight. Thus, the total
Answer (A) is incorrect because $800,000 is the amount of gross purchases. Answer (B) is remittance is $2,944 ($2,744 + $200).
incorrect because $806,000 incorrectly includes freight-out as a cost instead of freight-in.
Answer (D) is incorrect because $826,000 incorrectly includes freight-out. 34. P11,200 (P12,000 – P800)

26. Answer D is correct. The requirement is to determine the amount that Den should record as 35. The correct answer was B.
the cost of inventory as a result of the purchase. Inventory costs include all costs necessary to After removal of the returned merchandise, the company owes $24,000. The net method
prepare goods for sale. For a merchandising concern, these costs include the purchase price assumes that it is only normal and necessary to pay the amount due after the discount has
of the goods, freight-in, insurance, warehousing, and any costs necessary to get the goods to been taken (3 percent according to the terms). The discount here is $720 (3 percent of
the point of sale (except interest on loans obtained to purchase the goods). Therefore, the $24,000) so that the company only anticipates paying $23,280 ($24,000 less $720) which is
correct answer is (d) because inventory costs are equal to $4,925 ($3,750 + $175 + $900 + the amount recorded when the net method is applied. If the entire $24,000 is eventually paid,
$100). the additional $720 is recorded as a loss or as interest expense.

27. Answer is (D). 36. P70,560 (P100,000 x 80% x 90% x 98%)


Cost of purchases 5,000,000
Import duties 400,000 37. Answer B is correct. Purchases are always recorded net of trade discounts. When more than
Freight and insurance 1,000,000 one trade discount is applied to a list price, it is called a chain discount and is applied in steps,
Other handling costs 100,000 i.e., each discount applies to the previously discounted price:
Brokerage commission List price [$5,000 – ($5,000 x 20%)] = $4,000 Discounted price
200,000
Next, [$4,000 – ($4,000 x 10%)] = $3,600 Gross billing price
Total cost of purchases 6,700,000 When the net method is used the gross billing price must also be reduced by the cash
discount to get the amount to be recorded as purchases. The cash discount of 2% is then
28. $990 ($1,000 x 99%)
applied to the gross billing price: Under the gross method, purchases are recorded at their gross amount and offset by a
Gross billing price $3,600 purchase discounts account for discounts taken. Net purchases included in the determination
2% discount 72 of cost of goods available for sale are equal to the gross purchase amount of $ $90,000 less
Net purchase price $3,528 the $800 ($40,000 x 2%) discount taken. Net purchases equal $89,200 under the gross
Using the net method, the purchase is recorded at $3,528. method.
Answers (A), (B), and (D) are incorrect because the net method records purchases as if the
38. a ($9,500 × .99) = $9,405. discounts were taken, and the gross method records purchases to reflect the actual discounts
taken.
39. $6,000 x 98% = $5,880
46 . A Accounts payable 3,000,000
40. d $1,200 – ($1,200 × .02) = $1,176. Available discount ( 50,000)
Net amount 2,950,000
41. d ($16,000 – $1,200) × .02 = $296.
47. Answer (A) is correct. The gross method records purchases and accounts payable without
42. P9,600 [(328,000 + 90,000 – 8,000) x 3%] – 2,700 regard to purchase discounts available, for example, cash discounts for early payment. The
net method records purchases and accounts payable at the cash (discounted) price. If the
43. Answer is (B). accounts payable balance at the gross amount is $50,000 and $800 of discounts are available,
3,280,000 + 900,000– 80,000 = 4,100,000 x 3% =123,000; 123,000–27,000=96,000 the accounts payable balance at the net amount must be $49,200.
Answer (B) is incorrect because $49,100 results from assuming that purchase discounts of
44 . (B) Purchases of IBM compatibles 1,700,000 $900 are available. However, the $900 of discounts taken relates to accounts that have
Purchases of commercial software packages 1,200,000 already been paid. Answer (C) is incorrect because $47,900 assumes that purchase discounts
Total 2,900,000 of $2,100 are available. But discounts of only $800 are available, discounts of $900 were
Less: Returns and allowances 50,000 taken, and discounts of $1,300 were lost. Answer (D) is incorrect because $47,800 reduces
Net purchases 2,850,000 the accounts payable balance by the sum of discounts taken and lost. These discounts do not
Discounts available on purchases (2% x 2,850,000) 57,000 relate to the existing payables.
Less: Purchase discounts taken 17,000
Discount lost 40,000 48. REQUIRED: The accounts payable balance after the conversion.
DISCUSSION: (A) The gross method records purchases and accounts payable without regard
45. REQUIRED: The amounts at which net purchases should be valued under the net and gross to purchase discounts available, for example, cash discounts for early payment. The net
methods. method records purchases and accounts payable at the cash (discounted) price. If the
DISCUSSION: (C) The 2/15, net 30 credit phrase indicates that a 2% discount may be taken if accounts payable balance at the gross amount is $30,000 and $200 of discounts are available,
payment is made within 15 days of the invoice date and that payment is overdue if not made the accounts payable balance at the net amount must be $29,800.
within 30 days. Answer (B) is incorrect because $29,200 assumes that $800 of purchase discounts are
Under the net method, purchases are recorded net of any discount. Purchase discounts not available. However, the $800 of discounts taken relate to accounts that have already been
taken are reflected as an expense in purchase discounts lost. The $90,000 in purchases paid. Answer (C) is incorrect because $28,800 assumes that $1,200 of purchase discounts
should be recorded net of the 2% discount at $88,200 ($39,200 + $49,000) to determine cost are available. But only $200 of discounts are available, $800 of discounts were taken, and
of goods available for sale. $1,000 of discounts were lost. Answer (D) is incorrect because $28,200 reduces the accounts
payable balance by the sum of discounts taken and lost. There discounts do not relate to the
existing payables. the $90 cost of goods sold. Answer (C) is incorrect because $150 results from calculating
earnings before tax as .5 times net income instead of 2 times net income.
49 .(A) Accounts payable at gross 1,500,000
Discounts available in the accounts payable balance ( 10,000) 57. Answer is (C).
Accounts payable at net 1,490,000 .15/.25=60%; 100%-60%-10% - 15% - 3% = 12%; 480,000/12% = 4.0M

50. $2,250 ($1,350 + $450 x 2) 58. Answer (B) is correct. Cost of goods sold equals beginning inventory, plus purchases
(including freight-in), minus ending inventory. Given that sales reflect 125% of cost, cost of
51. a $360,000 + ($420,000 – $240,000) = $540,000. goods sold must equal 720,000 (900,000 sales ÷ 1.25). Consequently, the beginning inventory
must have been 85,000 (720,000 CGS + 80,000 EI - 690,000 purchases - 25,000 freight-in).
52. Answer (D) is correct. Net income before taxes is $20 [$10 NI ÷ (1.0 - .5 tax rate)]. Hence, the Answer (A) is incorrect because 40,000 is based on a 25% markup on sales. Answer (C) is
gross margin (sales - cost of sales) is $90 ($20 NI before taxes + $20 interest + $50 G&A incorrect because 110,000 omits the freight-in from the computation of cost of goods available
expenses). Sales must then be $180 ($90 gross margin ÷ 50% gross margin ratio). for sale. Answer (D) is incorrect because 265,000 uses the sales figure for cost of goods sold.
Answer (A) is incorrect because $90 is the cost of goods sold. Answer (B) is incorrect because
$135 is calculated by adding the 50% gross margin to 1 and multiplying the resulting 1.5 by 59. Answer is (C).
the $90 cost of goods sold. Answer (C) is incorrect because $150 results from calculating Merchandise inventory, beginning $11,000
earnings before tax as .5 times net income instead of 2 times net income. Add: Merchandise purchased ?
Goods available for sale ?
53. Choice "D" is correct. Deduct: Finished goods inventory, ending 17,000
Gross sales $1,000,000 Cost of goods sold $68,000
Sales returns ($1,000,000 x (7% + 3%) (100,000)
Net sales $ 900,000 Goods available for sale = $68,000 + $17,000
Goods available for sale = $85,000
54. c $15,720,000 – $370,000 – $175,000 = $15,175,000.
Merchandise purchased = $85,000 − Merchandise inventory, beginning
55. REQUIRED: The sales revenue for the year. Merchandise purchased = $85,000 − $11,000
DISCUSSION: (D) Net income before taxes is $100 [$70 NI  (1.0 – 0.3 tax rate)]. Hence, Merchandise purchased = $74,000
the gross margin (sales – cost of sales is $170 ($100 NI before taxes + $20 interest + $50
G&A expenses). Sales must then be $425 ($170 gross margin  40% gross margin ratio). 60. Choice "b" is correct. Using the BASE account analysis format, purchases can be squeezed
Answer (A) is incorrect because $170 is the gross margin. Answer (B) is incorrect because out as follows:
$255 is the cost of goods sold. Answer (C) is incorrect because $350 assumes pre-tax net Beginning Inventory (given) 30,000
income was $70. Purchases (squeezed) 208,000
Goods available for sale (added up) 238,000
56. Answer (D) is correct. Net income before taxes is $20 [$10 NI ÷ (1.0 - .5 tax rate)]. Hence, the COGS (275,000 x 1-.20) (220,000)
gross margin (sales - cost of sales) is $90 ($20 NI before taxes + $20 interest + $50 G&A Ending Inventory (given) 18,000
expenses). Sales must then be $180 ($90 gross margin ÷ 50% gross margin ratio).
Answer (A) is incorrect because $90 is the cost of goods sold. Answer (B) is incorrect because 61. Answer B is correct. To solve the problem, first calculate cost of sales. Since gross margin is
$135 is calculated by adding the 50% gross margin to 1 and multiplying the resulting 1.5 by 20%, cost of sales is equal to $220,000 ($275,000 x 80%). Then, purchases are calculated by
adding ending inventory and deducting beginning inventory from cost of sales. $208,000 CGS x 130%=5.2M
($18,000 – $30,000 + $220,000).
68. REQUIRED: The cost of goods available for sale.
62. 1,500,000 x .65 = 975,000(Cost of Goods SolD. DISCUSSION: (C) Because the gross margin equals 40% of net sales, cost of goods sold
975.000 + 350,000 = 1,325,000 (Goods Available) equals 60% of net sales, or $2,160,000. Cost of goods available for sale equals the cost of
1,325,000 - 300,000 = 1,025,000 (Purchases) goods sold plus the cost of the goods in ending inventory. Hence, cost of goods available for
sale equals $2,160,000 plus $240,000, or $2,400,000 (BI + PUR = GAFS* = CGS + EI).
63. Cost of Goods Sold = 55% x 60,000 = 33,000 Freight-in and purchase discounts are not used to estimate CGS or GAFS in the gross margin
Goods Available = 33,000 + 95,000 = 128,000 approach.
Purchases = 128,000 - 50,000 = 78,000 Ending inventory $ 240,000
Cost of goods sold 2,160,000
64. Cost of Goods Sold = 60% x 400,000 = 240,000 Goods available for sale $2,400,000
Goods Available = 80,000+240,000 = 320,000 Answer (A) is incorrect because $1,680,000 is gross margin plus ending inventory. Answer
Purchases = 320,000 - 140,000 = 180,000 (B) is incorrect because $1,920,000 is cost of goods sold minus ending inventory. Answer (D)
is incorrect because $2,440,000 is cost of goods available for sale plus freight-in and minus
65. Choice "B" is correct. Using the BASE account analysis format, purchases can be squeezed purchase discounts.
out as follows:
Beginning Inventory (given) 30,000 69. Answer C is correct. Gross margin is 40% of net sales ($1,800,000), or $720,000. Therefore,
Purchases (squeezed) 208,000 cost of goods sold is $1,080,000 ($1,800,000 net sales less $720,000 gross margin). Finally,
Goods available for sale (added up) 238,000 cost of goods available for sale is $1,200,000 ($1,080,000 cost of goods sold plus $120,000
COGS (275,000 x 1-.20)
 220,000 ending inventory). The amounts for freight-in ($45,000) and purchase discounts ($25,000) are
Ending Inventory (given) 18,000 not necessary for the computation.
66. Answer (B) is correct. Purchase discounts, allowances, and returns are subtractions from 70. (D) Beginning inventory $ 94,000
purchases because they are reductions of cost. Transportation-in is an addition because it Inventory purchased 400,000
increases cost. Thus, net purchases equals 346,000 (400,000 + 6,000 - 40,000 - 15,000 - Freight charges 6,000
5,000). Merchandise returned (5,000)
Answer (A) is incorrect because 340,000 omits transportation-in from the calculation. Answer Discounts [($400,000 - 5,000) x 1%)] (3,950)
(C) is incorrect because 370,000 omits transportation-in and adds, rather than subtracts, Ending inventory $491,050
purchase allowances. Answer (D) is incorrect because 376,000 adds, rather than subtracts,
purchase allowances. 71. Answer is (B).
Beginning inventory $ 94,000
67. Answer is (B). Inventory purchased 400,000
Cost of sales = 20/50 = 40%
 Freight 7,500
100%-40% = 60% - 20%-5% = 35% Merchandise returned (5,000)
Profit before tax
2,450,000/70% = 3.5M; Discounts [($400,000 – 5,000) x 1%] (3,950)
3.5M/35% = 10M; Cost of goods available for sale $492,550
10M x 40% = 4M
72. (B) Beginning inventory $ 62,000 $800,000 + $1,600,000 – $1,500,000 = $900,000.
Inventory purchased 280,000
Freight charges 9,000 80. Answer B is correct. When using the gross margin method of inventory valuation, the CGS is
Merchandise returned ( 4,000) estimated as Sales - (Sales x Gross margin). Silver Company's estimated CGS is $3,600,000
Discounts [($280,000 - 4,000) x 2%)] ( 5,520) [$4,800,000 – ($4,800,000 x .25)]. Therefore, ending inventory can be calculated as follows:
Cost of goods available for sale $341,480 Beginning inventory $ 900,000
Add: Purchases $3,400,000
73. Answer is (B). Freight-in 200,000 3,600,000
Beginning inventory $ 52,000 Cost of goods available 4,500,000
Inventory purchased 280,000 Deduct:
Freight 9,000 Cost of goods sold (estimated) ($3,600,000)
Merchandise returned (4,000) Ending inventory $ 900,000
Discounts [($280,000 – 4,000) x 2%] (5,520)
Cost of goods available for sale $331,480 81. (B) Beginning inventory $94,000
Inventory purchased 400,000
74. Answer is (D) Freight 6,000
Cost of goods sold (150,000 * 4) 600,000 Merchandise returned (5,000)
Ending Inventory 260,000 Discounts [($400,000 - 5,000) x 1%)] (3,950)
Cost goods available for sale 860,000 Cost of goods available for sale $491,050
Cost of goods sold 380,000
Ending inventory $111,050
75. Answer is (D).
150,000 x 8 = 1,200,000 + 80,000 = 1,280,000 82. (D) Beginning inventory $ 62,000
Inventory purchased 280,000
76. b COGS = $300,000 ÷ 1.25 = $240,000 Freight charges 9,000
($220,000 + $172,000 – $8,000) – $240,000 = $144,000. Merchandise returned (4,000)
Discounts [($280,000 - 4,000) x 2%)] (5,520)
77. Answer (B) is correct. The cost of goods sold equals 120,000 (180,000 sales - 60,000 gross Cost of goods available for sale 341,480
margin). Because cost of goods sold equals beginning inventory, plus purchases, minus Cost of goods sold 316,000
ending inventory, the amount of ending inventory must equal 100,000 (50,000 + 170,000 - Ending inventory $ 25,480
120,000).
Answer (A) is incorrect because 60,000 is the gross margin. Answer (C) is incorrect because 83. Answer is (B).
120,000 is the cost of goods sold. Answer (D) is incorrect because 160,000 deducts the gross Beginning inventory $ 94,000
margin from the goods available for sale. Inventory purchased 400,000
Freight 7,500
78. b $900,000 – ($800,000 × .80) = $260,000. Merchandise returned (5,000)
Discounts [($400,000 – 5,000) x 1%] (3,950)
79. c COGS = $2,000,000 × .75 = $1,500,000 Cost of goods available for sale $492,550
Cost of goods sold 380,000 DISCUSSION: (D) The entry to record cost of goods sold will be
Ending inventory $112,550 Cost of goods sold $446,000
Ending inventory 34,000
84. Answer is (B). Purchases $450,000
Beginning inventory $ 52,000 Beginning inventory 30,000
Inventory purchased 280,000 The debit to cost of goods sold often is made directly to an income summary account.
Freight 9,000 Answer (A) is incorrect because $450,000 is the amount of purchases. Answer (B) is incorrect
Merchandise returned (4,000) because $30,000 is the beginning inventory. Answer (C) is incorrect because $34,000 is the
Discounts [($280,000 – 4,000) x 2%] (5,520) ending inventory.
Cost of goods available for sale $331,480
Cost of goods sold 316,000 88. Answer is (C).
Ending inventory $ 15,480 Merchandise inventory, beginning $ 19,000
Add: Merchandise purchased 87,000
85. Answer is (B). Goods available for sale 106,000
Beginning inventory - 2015 2,355,000 Deduct: Finished goods inventory, ending 11,000
Purchases 3,180,000 Cost of goods sold $ 95,000
Freight in 220,000
Purchase discounts ( 45,000) 89. Beginning Inventory (11,200) + Purchases (32,200) = Goods Available (43,400) – Ending
Inventory (9,800) = Cost of Goods Sold (33,600)
Purchase returns (40,000)
Purchase allowances (15,000) 90. Answer is (B).
Goods available for sale 5,655,000 Inventory - December 31,2013 360,000
Cost of sales- 2015 (4,500,000 x 73%) 3,285,000 Purchases-2014 3,000,000
Ending inventory - 2015 2,370,000 Goods available for sale 3,360,000
Inventory - December 31,2014 ( 380,000)
Sales 100% Cost of goods sold 2,980,000
Cost of sales 73%
Gross profit rate 27% 91. The correct answer was C.
Cost of goods sold is determined by starting with the cost of beginning inventory and adding
86. Answer C is correct. The requirement is to determine the amount of shipping costs that should the cost of the merchandise purchased during the period before subtracting the cost of ending
be included in Seafood Trading’s year-end inventory valuation. The shipping costs to export to inventory. The issue here is what to include in the cost of the merchandise purchased. The
customers are a selling expense and not included in inventory. Shipping costs or freight-in basic rule is that all normal and necessary costs spent to get the inventory in position and
necessary to get the inventory in place to sell should be recorded in inventory. Seafood condition to be sold should be included as a capitalized cost. Thus, the freight cost to obtain
Trading should include a proportionate amount of the shipping costs of $1.5 million in ending the inventory is included but not the amount paid to deliver the merchandise to the customer.
inventory. Answer (c) is correct because $375,000 in shipping costs [($3.0 million ÷ $12.0 This second cost is viewed as a selling expense. Cost of goods sold is computed as beginning
million) × $1.5 million] should be included in the cost of ending inventory. inventory of $120,000 plus purchases of $359,000 (cost of $340,000 plus freight-in of $19,000)
to arrive at goods available for sale of $479,000. Subtract out the ending inventory that
87. REQUIRED: The net debit to cost of goods sold. remains of $165,000 which leaves cost of goods sold as $314,000.
Answer (A) is incorrect because $102,800 is the amount of gross purchases. Answer (C) is
92. REQUIRED: The cost of goods sold for the year. incorrect because $123,360 treats freight-out as a cost of goods sold. Answer (D) is incorrect
DISCUSSION: (B) Freight-in is the cost of receiving inventory and is a product cost. Freight- because $128,500 omits purchase discounts from the calculation.
out is the cost of shipping products to customers and should be treated as a selling expense
(period cost). Thus, CGS is $460,000. 96. Answer (B) is correct. Cost of goods sold equals beginning inventory, plus net purchases,
Beginning inventory $170,000 minus ending inventory. Given net purchases of 500,000, cost of goods sold equals 450,000
Purchases $450,000 (100,000 + 500,000 - 150,000).
Freight-in 50,000 500,000 Answer (A) is incorrect because 250,000 subtracts, rather than adds, beginning inventory.
Goods available $670,000 Answer (C) is incorrect because 550,000 subtracts, rather than adds, opening inventory and
Ending inventory (210,000) adds, rather than subtracts, closing inventory. Answer (D) is incorrect because 750,000 adds,
Cost of goods sold $460,000 rather than subtracts, closing inventory.
Answer (A) is incorrect because freight out should be excluded from ending inventory. Answer
(C) is incorrect because freight-in, not freight-out, should be included in the cost of goods 97. REQUIRED: The cost of goods sold for the year.
available. Answer (D) is incorrect because freight-out should not be added to the cost of DISCUSSION: (C) As indicated in the T-account analysis below, cost of goods sold equals
goods available. purchases plus any decrease in inventory or minus any increase in inventory (purchases
minus the change in inventory). The write-off of obsolete inventory is a loss, not a component
93 . Beginning inventory 1,700,000 of CGS. Thus, cost of goods sold is $300,000.
Purchases 4,500,000 Inventory
Freight in 500,000 12/31/00 $180,000 $ 68,000 obsolescence
Goods available for sale 6,700,000 Purchases 248,000 300,000 CGS
Less: Ending inventory 2,100,000 $ 60,000
Cost of goods sold 4,600,000 Answer (A) is incorrect because $436,000 results from adding obsolete inventory to, not
subtracting it from, beginning inventory. Answer (B) is incorrect because $368,000 includes
Freight out is a selling expense. the obsolete inventory in CGS. Answer (D) is incorrect because $248,000 equals purchases.

94. Answer is (A). 98. (c) To compute cost of goods sold, the solutions approach is to set up a T-account for
Beginning inventory 160.000 inventory
Purchases 530.0,00 Inventory
Purchase discounts ( 10,000) 12/31/02 90 ,000
Goods available for sale 680,000 Purchases 124,000 34,000 Write-off
Ending inventory (215,000) ? Cost of goods sold
Cost of goods sold 465.000 12/31/03 30,000
Purchases increase inventory, while the write-off and cost of goods sold decrease inventory.
95. REQUIRED: The cost of goods sold reported for the year. Cost of goods sold can be computed as $150,000 using the T-account. An alternate solutions
DISCUSSION: (B) Cost of goods sold equals beginning inventory, plus net purchases, plus approach is to use the CGS computation
freight-in, minus ending inventory. Freight-out is a cost of selling the goods rather than a cost BI $ 90,000
of acquiring the goods. Thus, cost of goods sold is $118,220 [$30,840 + ($102,800 – $10,280) + Purchases 124,000
+ $15,420 – $20,560]. CGAS 214,000
- EI 30,000 purchase allowances. Answer (D) is incorrect because 376,000 adds, rather than subtracts,
$184,000 purchase allowances.
Accounted for as follows:
$ 34,000 recognized as invy. loss* 102. Answer (B) is correct. Cost of goods sold equals beginning inventory, plus net purchases,
$150,000 recognized as CGS minus ending inventory. Given net purchases of 500,000, cost of goods sold equals 450,000
* Theoretically correct treatment. (100,000 + 500,000 - 150,000).
Answer (A) is incorrect because 250,000 subtracts, rather than adds, beginning inventory.
99 . Inventory – 12/31/2003 360,000 Answer (C) is incorrect because 550,000 subtracts, rather than adds, opening inventory and
Purchases – 2004 3,000,000 adds, rather than subtracts, closing inventory. Answer (D) is incorrect because 750,000 adds,
Goods available for sale 3,360,000 rather than subtracts, closing inventory.
Inventory – 12/31/2004 380,000
Cost of goods sold 2,980,000 103. c $650,000 – ($650,000 ÷ 1.20) = $108,333.

The physical inventory of P380,000 rather than the perpetual inventory of P420,000, is 104. Answer D is correct. Gross margin is sales minus cost of goods sold and is computed as
considered in computing cost of goods sold because the company’s policy is to indicate follows for this question:
inventory shortages in cost of goods sold. Sales $150,000
Less cost of goods sold
100. Choice "B" is correct, $512,000 cost of goods sold on the income statement. Beginning inventory $ 35,000
Rule: Consignor must include consigned goods (in the hands of the consignee) in his own Purchases 70,000
inventory, at his cost plus warehousing costs of consignor before goods are transferred to Goods available $105,000
consignee plus shipping costs to consignee.
 Ending inventory (50,000)
Beginning inventory $122,000 Cost of goods sold (55,000)
Add (deduct): Gross margin $ 95,000
Purchases 540,000 Note that the $5,000 of sales commissions are not included in the calculation of cost of goods
Freight in 10,000 sold. This is because cost of goods sold includes only those costs associated with bringing
Transportation to consignees 5,000 goods to the point of sale and converting the goods into a salable condition. Sales
Cost of goods available for sale 677,000 commissions do not fit this definition because sales commissions are a cost from the point of
Ending inventory sale.
Held by Kam (145,000)
Held by consignees (20,000) 105. Choice "a" is correct. $190,000.
Cost of goods sold $512,000 Sales: $300,000
Less: Cost of sales:
101. Answer (B) is correct. Purchase discounts, allowances, and returns are subtractions from Beginning inventory Jan. 1 $ 70,000
purchases because they are reductions of cost. Transportation-in is an addition because it Add: Purchases 140,000
increases cost. Thus, net purchases equals 346,000 (400,000 + 6,000 - 40,000 - 15,000 - Subtotal 210,000
5,000). Less: Ending inventory (100,000)
Answer (A) is incorrect because 340,000 omits transportation-in from the calculation. Answer Cost of sales (110,000)
(C) is incorrect because 370,000 omits transportation-in and adds, rather than subtracts, Gross margin $190,000 A
Note: The sales commissions of $10,000 is a selling expense. 112. c 25% ÷ (100% – 25%) = 33%.

106. c $100,000 – $2,000 - $60,000 = $38,000. 113. C


Income before tax $500,000
107. c $150,000 – $4,000 - $90,000 = $56,000. Income tax ($500,000 x 40%) (200,000)
Net income $300,000
108. Choice "C" is correct. Gross profit is calculated as sales less cost of goods sold.
Net sales $500,000 114. Answer is (A).
Cost of sales 100,000 Sales 9,600,000
Gross profit $400,000 Cost of sales (9,600,000 / 160%) 6,000,000
Choice "A" is incorrect. Insurance expense, wages and liabilities are not subtracted to arrive at Gross income 3,600,000
gross profit. Choice "B" is incorrect. Insurance expense and wages are not part of cost of Distribution & administrative expenses (30% x 9,600,000) 2,880,000
goods sold. $275,000 is the amount of net income. Choice "D" is incorrect. Cost of sales is Net income 720,000
subtracted from sales to arrive at gross profit.
115. Answer is (B).
109. Answer is (B). Cost of goods sold (10% / 25%) 40%
.125/.25 = .50; 100% - 50% - 12.5% - 17.5% - 5% = 15% 750,000/15% = 5,000,000 x 50% =
2,500,000 Sales (2,000,000/40%) 5,000,000
Sales discount ( 100,000)
110. Answer is (). Net sales 4,900,000
Sales 6,200,000 Cost of goods sold (2,000,000)
Less: Sales returns 200,000 Gross profit 2,900,000
Net sales 6,000,000 Administrative expenses (10% x 5,000,000) (500,000)
Cost of sales: Distribution costs (2,000,000)
Interest expense (20,000)
Inventory – January 1,000,000
Income before income tax 380,000
Purchases 5,500,000
Freight-in 250,000 Operating expenses ( 500,000 /1/5) 2,500,000
Total 5,750,000 Administrative expenses (500,000)
Less: Purchase returns, allow. & discounts 150,0005,600,000 Distribution costs 2,000,000
Goods available for sale 6,600,000
Less: Inventory – December 31 2,100,000 4,500,000 116. Answer is (C)
Gross income 1,500,000 3,500,000/70% = 5,000,000
Gross profit rate (1,500,000 / 4,500,000) 33 1/3%
117. Answer is (C)
.40 5M-3.5M=1.5M – (60% x 1.5M) = 600,000
111. a  2.9  29%
1  .40
118. Answer is (B).
3,500,000 – 500,000 = 3,000,000 1, and no cash outlay is required at year-end. Answer (C) is incorrect because accrued
interest is $50,000. Also, interest expense and interest payable is debited credit.
119. Answer (B) is correct. Because the tax basis uses an accelerated method, depreciation
expense and accumulated depreciation will be greater. Moreover, taxable income will be lower 123. Answer (A) is correct. Current period pre-tax net income equals $280,000 ($750,000 sales -
than financial net income. Consequently, tax basis retained earnings will be less than that in $200,000 CGS - $60,000 depreciation - $10,000 interest - $200,000 administrative expenses).
the general purpose financial statements. Thus, after-tax net income credited to retained earnings equals $140,000 [(1.0 - .5) x
Answer (A) is incorrect because the balance of accumulated depreciation is higher in the tax $280,000].
basis financial statements. Answer (C) is incorrect because the historical cost of fixed assets Answer (B) is incorrect because income taxes are omitted and the journal entry is reversed.
is recorded in the gross fixed assets account. This amount is unaffected by depreciation. Answer (C) is incorrect because administrative expenses were omitted. Answer (D) is incorrect
Answer (D) is incorrect because the accounts receivable balance is unaffected by the because administrative expenses were omitted and the journal entry is reversed.
depreciation method used.

120. Answer (A) is correct. Cost of goods sold equals beginning inventory, plus purchases, minus
ending inventory. Hence, cost of goods old is $440,000 ($140,000 + $530,000 - $230,000).
Answer (B) is incorrect because $530,000 equals purchases. Answer (C) is incorrect because
$620,000 is obtained by reversing the opening and closing inventory figures. Answer (D) is
incorrect because $670,000 omits closing inventory from the calculation.

121. Answer (B) is correct. The year-end total assets can be determined by summing all of the
assets and deducting accumulated depreciation (including the current year's depreciation).
Total accumulated depreciation at the end of the second year is $120,000 [($600,000 ÷ 10
years) x 2 years]. Total assets equal $890,000 ($80,000 cash + $100,000 A/R + $230,000 EI +
$600,000 gross fixed assets - $120,000 accumulated depreciation).
Answer (A) is incorrect because $800,000 uses the beginning balance of inventory. Answer
(C) is incorrect because $950,000 omits second-year depreciation from the calculation.
Answer (D) is incorrect because $1,010,000 omits total accumulated depreciation from the
calculation.

122. Answer (D) is correct. The debt was issued on July 1 and has only been outstanding for 6
months. Interest expense equals the face amount of the debt multiplied by the interest rate
and the fraction of the year ($1,000,000 x 10% x 6/12 = $50,000). Because interest is payable
on July 1, 6 months' interest is accrued and expensed in the current period. The payable is
also recognized in the current period. Thus, the adjusting entry should be
Interest expense $50,000
Interest payable $50,000
Answer (A) is incorrect because the debt has been outstanding for only 6 months so accrued
interest is only $50,000. Answer (B) is incorrect because the debt pays annual interest on July

S-ar putea să vă placă și